06.10.2024

Зависимость сопротивления проводника от длины: Зависимость электрического сопротивления от сечения, длины и материала проводника

Содержание

Зависимость электрического сопротивления от сечения, длины и материала проводника

  

Сопротивление различных проводников зависит от материала, из которого они изготовлены.

Можно проверить это практически на следующем опыте.

Рисунок 1. Опыт, показывающий зависимость электрического сопротивления от материала проводника

Подберем два или три проводника из различных материалов, возможно меньшего, но одинакового поперечного сечения, например, один медный, другой стальной, третий никелиновый. Укрепим на планке два зажима а и б на расстоянии 1 —1,5 м один от другого (рис. 1) и подключим к ним аккумулятор через амперметр. Теперь поочередно между зажимами а и б будем на 1—2 сек включать сначала медный, потом стальной и, наконец, никелиновый проводник, наблюдая в каждом случае за отклонением стрелки амперметра. Нетрудно будет заметить, что наибольший по величине ток пройдет по медному проводнику, а наименьший — по никелиновому.

Из этого следует, что сопротивление медного проводника меньше, чем стального, а сопротивление стального проводника меньше, чем никелинового.

Таким образом, электрическое сопротивление проводника зависит от материала, из которою он изготовлен.

Для характеристики электрического сопротивления различных материалов введено понятие о так называемом удельном сопротивлении.

Определение: Удельным сопротивлением называется сопротивление проводника длиной в 1 м и сечением в 1 мм2 при температуре +20 С°.

Удельное сопротивление обозначается буквой ρ («ро») греческого алфавита.

Каждый материал, из которого изготовляется проводник, обладает определенным удельным сопротивлением. Например, удельное сопротивление меди равно 0,0175 Ом*мм2/м, т. е. медный проводник длиной 1 м и сечением 1 мм2 обладает сопротивлением 0,0175 Ом.

Ниже приводится таблица удельных сопротивлений материалов, наиболее часто применяемых в электротехнике.

Удельные сопротивления материалов, наиболее часто применяемых в электротехнике















МатериалУдельное сопротивление,  Ом*мм2
 Серебро0,016
 Медь0,0175 
 Алюминий0,0295 
 Железо0,09-0,11
 Сталь0,125-0,146
 Свинец0,218-0,222
 Константан0,4-0,51
 Манганин0,4-0,52
 Никелин0,43
 Вольфрам0,503
 Нихром1,02-1,12
 Фехраль1,2
 Уголь10-60

Любопытно отметить, что например, нихромовый провод длиною 1 м обладает примерно таким же сопротивлением, как медный провод длиною около 63 м (при одинаковом сечении).

Разберем теперь, как влияют размеры проводника, т. е. длина и поперечное сечение, на величину его сопротивления.

Воспользуемся для этого схемой, изображенной на рис. 1. Включим между зажимами а и б для большей наглядности опыта проволоку из никелина. Заметив показание амперметра, отключим от зажима б проводник, которой соединяет прибор с минусом аккумулятора, и освободившимся концом проводника прикоснемся к никелиновой проволоке на некотором удалении от зажима а (рис. 2). Уменьшив таким образом длину проводника, включенного в цепь, нетрудно заметить по показанию амперметра, что ток в цепи увеличился.

 

Рисунок 2. Опыт, показывающий зависимость электрического сопротивления от длины проводника

Это говорит о том, что с уменьшением длины проводника сопротивление его уменьшается. Если же перемещать конец проводника по никелиновой проволоке вправо, т. е. к зажиму б, то, наблюдая за показаниями амперметра, можно сделать вывод, что с увеличением длины проводника сопротивление его увеличивается.

Таким образом, сопротивление проводника прямо пропорционально его длине, т. е. чем длиннее проводник, тем больше его электрическое сопротивление..

Выясним теперь, как зависит сопротивление проводника от его поперечного сечения, т. е. от толщины.

Подберем для этого два или три проводника из одного и того же материала (медь, железо или никелин), но различного поперечного сечения и включим их поочередно между зажимами а и б, как указано на рис. 1.

Наблюдая каждый раз за показаниями амперметра, можно убедиться, что чем тоньше проводник, тем меньше ток в цепи, а следовательно, тем больше сопротивление проводника. И, наоборот, чем толще проводник, тем больше ток в цепи, а следовательно, тем меньше сопротивление проводника.

Значит, сопротивление проводника обратно пропорционально площади его поперечного сечения, т. е. чем толще проводник, тем его сопротивление меньше, и, наоборот, чем тоньше проводник, тем его сопротивление больше.

Чтобы лучше уяснить эту зависимость, представьте себе две пары сообщающихся сосудов (рис. 3), причем у одной пары сосудов соединяющая трубка тонкая, а у другой — толстая.

Рисунок 3. Вода по толстой трубке перейдет быстрее, чем по тонкой

Ясно, что при заполнении водой одного из сосудов (каждой пары) переход ее в другой сосуд по толстой трубке произойдет гораздо быстрее, чем по тонкой. Это значит, что толстая трубка окажет меньшее сопротивление течению воды. Точно так же и электрическому току легче пройти по толстому проводнику, чем по тонкому, т. е. первый оказывает ему меньшее сопротивление, чем второй.

Обобщая результаты произведенных нами опытов, можно сделать следующий общий вывод:

 электрическое сопротивление проводника равно удельному сопротивлению материала, из которого этот проводник сделан, умноженному на длину проводника и деленному на площадь его поперечного сечения. .

Математически эта зависимость выражается следующей формулой:

 

где R—сопротивление проводника в Ом;

ρ — удельное сопротивление материала в Ом*мм2/м;

l — длина проводника в м;

S—площадь поперечного сечения проводника в мм2.

Примечание. Площадь поперечного сечения круглого проводника вычисляется по формуле

где π—постоянная величина, равная 3,14;

d—диаметр проводника.

Указанная выше зависимость дает возможность определить длину проводника или его сечение, если известны одна из этих величин и сопротивление проводника.

Так, например, длина проводника определяется по формуле:

Если же необходимо определить площадь поперечного сечения проводника, то формула принимает следующий вид:

Решив это равенство относительно ρ, получим выражение для определения удельного сопротивления проводника:

Последней формулой приходится пользоваться в тех случаях, когда известны сопротивление и размеры проводника, а его материал неизвестен и к тому же трудно определим по внешнему виду. Определив по формуле удельное сопротивление проводника, можно найти  материал, обладающий таким удельным сопротивлением.  

ПОНРАВИЛАСЬ СТАТЬЯ? ПОДЕЛИСЬ С ДРУЗЬЯМИ В СОЦИАЛЬНЫХ СЕТЯХ!

Похожие материалы:

Добавить комментарий

Расчет сопротивления провода по сечению, диаметру, длине

В своей работе электрик часто сталкивается с вычислением различных величин и преобразований. Так для корректного подбора кабеля приходится подбирать нужное сечение. Логика выбора сечения основана на зависимости сопротивления от длины линии и площади сечения проводника. В этой статье мы рассмотрим, как выполняется расчет сопротивления провода по его геометрическим размерам.

Формула для расчета

Любые вычисления начинаются с формулы. Основной формулой для расчета сопротивления проводника является:

R=(ρ*l)/S

Где R – сопротивление в Омах, ρ – удельное сопротивление, l – длина в м, S – площадь поперечного сечения провода в мм2.

Эта формула подходит для расчета сопротивления провода по сечению и длине. Из неё следует, что в зависимости от длины изменяется сопротивление, чем длиннее – тем больше. И от площади сечения – наоборот, чем толще провод (большое сечение), тем меньше сопротивление. Однако непонятной остаётся величина, обозначенная буквой ρ (Ро).

Удельное сопротивление

Удельное сопротивление – это табличная величина, для каждого металла она своя. Она нужна для расчета и зависит от кристаллической решетки металла и структуры атомов.

 

Из таблицы видно, что самое меньшее сопротивление у серебра, для медного кабеля оно равняется 0,017 Ом*мм2/м. Такая размерность говорит нам, сколько приходится Ом при сечении в 1 миллиметр квадратный и длине в 1 метр.

Кстати, серебряное покрытие используется в контактах коммутационных аппаратов, автоматических выключателей, реле и прочего. Это снижает переходное контактное сопротивление, повышает срок службы и уменьшает нагрев контактов. При этом в контактах измерительной и точной аппаратуры используют позолоченные контакты из-за того, что они слабо окисляются или вообще не окисляются.

У алюминия, который часто использовался в электропроводке раньше, сопротивление в 1,8 раза больше чем у меди, равняется 2,82*10-8 Ом*мм2/м. Чем больше сопротивление проводника, тем сильнее он греется. Поэтому при одинаковом сечении алюминиевый кабель может передать меньший ток, чем медный, это и стало основной причиной почему все современные электрики используют медную электропроводку. У нихрома, который используется в нагревательных приборах оно в 100 раз больше чем у меди 1,1*10-6 Ом*мм2/м.

Расчет по диаметру

На практике часто бывает так, что площадь поперечного сечения жилы не известна. Без этого значения ничего рассчитать не получится. Чтобы узнать её, нужно измерить диаметр. Если жила тонка, можно взять гвоздь или любой другой стержень, намотать на него 10 витков провода, обычной линейкой измерить длину получившейся спирали и разделить на 10, так вы узнаете диаметр.

Ну, или просто замерить штангенциркулем. Расчет сечения выполняется по формуле:

 

Обязательны ли расчеты?

Как мы уже сказали, сечение провода выбирают исходя из предполагаемого тока и сопротивления металла, из которого изготовлены жилы. Логика выбора заключается в следующем: сечение подбирают таким способом, чтобы сопротивление при заданной длине не приводило к значительным просадкам напряжения. Чтобы не проводить ряд расчетов, для коротких линий (до 10-20 метров) есть достаточно точные таблицы:

В этой таблице указаны типовые значения сечения медных и алюминиевых жил и номинальные токи через них. Для удобства указана мощность нагрузки, которую выдержит эта линия. Обратите внимание на разницу в токах и мощности при напряжении 380В, естественно, что это предполагается трёхфазная электросеть.

Напоследок рекомендуем просмотреть видео, на котором подробно рассказывается, как рассчитать сечение проводника, а также предоставлены примеры расчетных работ:

Расчет сопротивления провода сводится к использованию пары формул, при этом вы можете скачать готовые калькуляторы из Плэй Маркета для своего смартфона, например, «Electrodroid» или «Мобильный электрик». Эти знания пригодятся для расчетов нагревательных приборов, кабельных линий, предохранителей и даже популярных на сегодняшний день спиралей для электронных сигарет.

Материалы по теме:

Зависимость сопротивления от длины провода. Электрическое сопротивление проводника

Электрическое
сопротивление проводника:

1)
величина, характеризующая противодействие
проводника или электрической цепи
электрическому току;

2)
структурный элемент электрической
цепи, включаемый в цепь для ограничения
или регулирования силы тока.

Электрическое
сопротивление металлов з

ависит
от материала проводника, его длины и
поперечного сечения, температуры и
состояния проводника (давления,
механических сил растяжения и сжатия,
т. е. внешних факторов, влияющих на
кристаллическое строение металлических
проводников).

Зависимость
сопротивления от материала, длины и
площади поперечного сечения проводника:

где

— удельное сопротивление проводника;

l
– длина проводника;

S – площадь
поперечного сечения проводника.

Зависимость
сопротивления проводника от температуры:

или

,

где
R t
– сопротивление при температуре t
0 C;

R 0
– сопротивление при 0 0 C;


температурный коэффициент сопротивления,
который показывает, как изменяется
сопротивление проводника по отношению
к его сопротивлению при 0 0 C,
если температура изменяется на один
градус;

T – термодинамическая
температура.

Соединения
сопротивлений:

последовательное, параллельное,
смешанное.

а)
Последовательное соединение сопротивлений


представляет собой систему проводников
(сопротивлений), которые включены один
за другим, так что через каждое из
сопротивлений протекает один и тот же
ток:

I
= I 1
= I 2
==
I n .

Напряжение
при последовательном соединении
сопротивлений


равно сумме напряжений на каждом из
сопротивлений:

.

Напряжение
на каждом из последовательно соединенных
сопротивлений


пропорционально значению данного
сопротивления:

.

Распределение
напряжения по последовательно соединенным
элементам цепи (делитель напряжения)

:

,

U
– напряжение на участке цепи с
сопротивлением R 1 ;

R
– полное сопротивление соединения;

R 1
– сопротивление участка цепи с выбранным
сопротивлением.


равно
сумме отдельно взятых сопротивлений
и оно больше наибольшего из включенных:

.

Общее
сопротивление цепи при последовательном
соединении

n


одинаковых сопротивлений

:

,

где
n
– число сопротивлений, включенных
последовательно;

R 1
= значение отдельно взятого сопротивления.

б)
Параллельное соединение сопротивлений:


признаком такого соединения является
разветвление тока I на отдельные токи
через соответствующие сопротивления.
При этом ток I равен сумме токов через
отдельно взятое сопротивление:

.

Общее
напряжение при параллельном соединении


равно напряжению на отдельно взятом
сопротивлении:

U
= U 1
= U 2
= =
U i .

Связь
между током и сопротивлением при
параллельном соединении:


при параллельном соединении сопротивлений
токи в отдельных проводниках обратно
пропорциональны их сопротивлениям:

.

Величина,
обратная полному сопротивлению цепи
(общая проводимость) при параллельном
соединении,


равна сумме проводимостей отдельно
взятых проводников. При этом общее
сопротивление цепи меньше наименьшего
сопротивления из включенных:

;

.

Общая
проводимость цепи при параллельном
соединении

n


проводников:

G пар
= nG 1 ,

где
G пар
– проводимость цепи;

G 1
– проводимость отдельного взятого
проводника.

Шунтирование
электроизмерительных приборов


– расширение предела измерения тока
с помощью электроизмерительного
прибора, к которому присоединяют
параллельно проводник с малым
сопротивлением (шунт). В этом случае

,

где
I п
– ток, протекающий через прибор;

I
– ток в цепи;

n
= R п /R ш
– отношение сопротивления прибора R п
к сопротивлению шунта R ш.

Добавочное
сопротивление


– сопротивление, которое присоединяют
последовательно к электроизмерительному
прибору для расширения предела измерения
напряжения. При этом

,

где
U п
– напряжение на приборе;

U
– напряжение в цепи;

N
= R д /R п
– отношение величины добавочного
сопротивления к сопротивлению прибора.

Электрическая
проводимость

– физическая величина, обратная
сопротивлению проводника:

.

Сверхпроводимость

– свойство многих проводников, состоящее
в том, что их электрическое сопротивление
скачком падает до нуля при охлаждении
ниже определенной критической температуры
T k ,
характерной для данного материала.

Связь
удельной проводимости с удельным
сопротивлением (удельным электрическим
сопротивлением)

:

;

.

Зависимость
удельного сопротивления проводника
от температуры
:

,

где
 t
– удельное сопротивление при температуре
t
0 C;

 0
– удельное сопротивление при 0 0 C;


температурный коэффициент сопротивления,
который показывает, как изменяется
удельное сопротивление проводника по
отношению к его удельному сопротивлению
при 0 0 C,
если температура изменяется на один
градус.

Задания:

1.
Ознакомиться с применяемыми в работе
электроизмерительными приборами.
Результаты занести в табл. 1.

Таблица 1.

2.
Измерить удельное электрическое
сопротивление.

1.
Измерить микрометром в нескольких
местах рабочей части проводника его
диаметр. Рассчитать среднее значение
диаметра.

2.
Установить подвижный контакт на 0,5 
0,7 от длины рабочей части проводника.
Занести значение длины в таблицу 2.

3.
Включить установку в сеть переменного
тока с напряжением 220 В. При этом должна
загореться индикаторная лампочка.

4.
Провести измерения тока и напряжения.
Результаты занести в таблицу 2.

Таблица 2.

5.
Отключить установку. Установить
подвижный контакт на другое значение
рабочей части исследуемого проводника.
Вновь включить установку и определить
новые значения тока и напряжения.

Примечание.

Изменение
длины рабочей части проводника,
определение тока и напряжения проводятся
3-5 раз.

6.
Так как

,

,
(1)

где

— удельное электросопротивление
проводника;

ℓ —
длина проводника;

S
— площадь поперечного сечения.

,
(2)

где

— погрешность вольтметра;


приборная погрешность миллиамперметра;

 —
задается преподавателем;

d,
ℓ
— определяются известными методами.

10.
Записать полученный результат в виде
доверительного интервала

То как влияет материал проводника учитывается при помощи удельного сопротивления, которое принято обозначать буквой греческого алфавита ρ
и являет собой сопротивление проводника
сечением 1 мм 2 и длинной 1 м. У серебра наименьшее удельное сопротивление ρ = 0,016 Ом.мм 2 /м. Ниже приводятся значения удельного сопротивления
для нескольких проводников:

  • Сопротивление провода для серебра — 0,016,
  • Сопротивление провода для свинеца — 0,21,
  • Сопротивление провода для меди — 0,017,
  • Сопротивление провода для никелина — 0,42,
  • Сопротивление провода для люминия — 0,026,
  • Сопротивление провода для манганина — 0,42,
  • Сопротивление провода для вольфрама — 0,055,
  • Сопротивление провода для константана — 0,5,
  • Сопротивление провода для цинка — 0,06,
  • Сопротивление провода для ртути — 0,96,
  • Сопротивление провода для латуни — 0,07,
  • Сопротивление провода для нихрома — 1,05,
  • Сопротивление провода для стали — 0,1,
  • Сопротивление провода для фехрали -1,2,
  • Сопротивление провода для бронзы фосфористой — 0,11,
  • Сопротивление провода для хромаля — 1,45

Так как в состав сплавов входят разные количества примесей, то удельное сопротивление может изменятся. 2

  • где d — это диаметр провода.

Измерить диаметр провода можно микрометром либо штангенциркулем,но если их нету под рукой,то можно плотно намотать на ручку (карандаш) около 20 витков провода, затем измерить длину намотанного провода и разделить на количество витков.

Для определения длинны провода,которая нужна для достижения необходимого сопротивления,можно использовать формулу:

l=(S?R)/ρ

Примечания:

1.Если данные для провода отсутствуют в таблице,то берется некоторое среднее значение.Как пример,провод из никелина который имеет диаметр 0,18 мм площадь сечения равна приблизительно 0,025 мм2, сопротивление одного метра 18 Ом, а допустимый ток 0,075 А.

2.Данные последнего столбца,для другой плотности тока, необходимо изменить. Например при плотности тока 6 А/мм2, значение необходимо увеличить вдвое.

Пример 1
. Давайте найдем сопротивление 30 м медного провода диаметром 0,1 мм.

Решение
. С помощью таблицы берем сопротивление 1 м медного провода, которое равно 2,2 Ом. Значит, сопротивление 30 м провода будет R = 30.2,2 = 66 Ом.

Расчет по формулам будет выглядеть так: площадь сечения: s= 0,78.0,12 = 0,0078 мм2. Поскольку удельное сопротивление меди ρ = 0,017 (Ом.мм2)/м, то получим R = 0,017.30/0,0078 = 65,50м.

Пример 2
. Сколько провода из манганина у которого диаметр 0,5 мм нужно чтобы изготовить реостат, сопротивлением 40 Ом?

Решение
. По таблице выбираем сопротивление 1 м этого провода: R= 2,12 Ом: Чтобы изготовить реостат сопротивлением 40 Ом, нужен провод, длина которого l= 40/2,12=18,9 м.

Расчет по формулам будет выглядеть так. Площадь сечения провода s= 0,78.0,52 = 0,195 мм 2 . Длина провода l = 0,195.40/0,42 = 18,6 м.


При проектировании электросхем важно правильно выбрать материал и сечение проводов. Чаще всего для этих целей применяется медь, обладающая меньшим сопротивлением.

От чего зависит сопротивление металла

Электрический ток – это направленное движение заряженных частиц. В металлах это свободные электроны. Они двигаются между атомами кристаллической решётки. Сопротивление их движению зависит от металла или сплава, а также его температуры – при её повышении сопротивление провода электрическому току растёт.

Исключение составляют специальные сплавы, применяемые в измерительных приборах. Из них изготавливаются резисторы, не меняющие своих параметров при изменении температуры. Кроме того, для подключения термопар применяются двухжильные провода, сопротивление одного из которых при повышении температуры растёт, а другого – уменьшается. В результате параметры кабеля не меняется.

Удельное сопротивление различных металлов

Разные металлы обладают различными свойствами и используются для разных целей.

Медь и алюминий

Самыми распространёнными проводами являются медные и алюминиевые. У меди ниже электросопротивление, чем сопротивление алюминиевого провода, кабеля из неё имеют меньшее сечение. Она прочнее, это позволяет сделать кабеля тоньше, а также гибкими и многожильными. Кроме того, медь паяется оловянными припоями.

Но у алюминия есть одно преимущество: он намного дешевле. Поэтому его используют для намотки трансформаторов и прокладки проводки, при эксплуатации которой отсутствуют изгибы, движение или вибрация.

Другие металлы

  • Золото. Имеет самое малое электросопротивление, но из-за его цены используется только в отдельных местах в военной и космической технике;
  • Серебро. Обладает лучшим соотношением цена/качество, чем золото, но также применяется ограниченно, в основном для изготовления контактов и разъёмов – оно не окисляется;
  • Нихром (сплав никеля и хрома) и фехраль (железо, хром и алюминий). Обладают высокой температурой плавления. Сопротивление нихрома и нихромовой проволоки достаточно большое для изготовления нагревателей и проволочных сопротивлений;
  • Вольфрам. Имеет высокое удельное сопротивление и очень тугоплавкий – 3422 градуса. Из него изготавливаются нити накала в электролампочках;
  • Константан. Сплав из меди, никеля и марганца, не меняющий своих свойств при изменениях температуры. Применяется для изготовления резисторов в измерительных приборах;
  • Компенсационные. Из этих сплавов изготавливаются кабеля для подключения термопар и других датчиков. При повышении температуры электросопротивление одного проводника увеличивается, а другого – уменьшается. В результате общее значение остаётся неизменным.

Интересно.
В 50-е годы проектировались трансформаторы для высоковольтных подстанций с серебряными обмотками. С учётом пониженных потерь это было выгодно. Но из-за повышения цены на серебро на мировом рынке эти проекты не были реализованы.

Выбор сечения кабелей

При расчёте сечения токопроводящей жилы учитываются нагрев и падение напряжения в кабелях большой длины. Выполнить расчет сопротивления провода можно по специальным таблицам или при помощи онлайн-калькуляторов.

Сечение, рассчитанное по потерям, может быть больше или меньше рассчитанного по нагреву. Это зависит от длины кабеля. Для прокладки выбирается большее значение.

Выбор сечения проводника по допустимому нагреву

При протекании электрического тока по кабелю он греется. Этот нагрев может расплавить изоляцию, что приведёт к её разрушению и замыканию рядом расположенных проводов между собой или на заземлённые детали конструкций.

Важно!
Разрушение изоляции и К.З. (короткое замыкание) могут привести к пожару.

Для того чтобы предотвратить подобную ситуацию, сечение кабеля должно соответствовать току нагрузки, типу изоляции и условиям прокладки. По проводам, проложенным открыто, или с термостойкой изоляцией можно пропускать больший ток, чем по кабелю, проложенному по трубам в виниловой или резиновой оболочке.

Выбор сечения по потерям напряжения

При протекании электрического тока по кабелю происходит уменьшение напряжения возле нагрузки. Это связано с тем, что, хотя и сопротивление небольшого куска провода, и падение напряжения на нём невелико, на большой длине оно может достичь значительной величины.

Например, удельное сопротивление медного провода – 0,017 Ом мм²/м. Но в одножильном кабеле длиной 100 м сечением 10 мм² оно составит 0,17Ом. При токе 80А (допустимому по нагреву) падение напряжения в сети 220В составит 27В (100 м фазного провода и 100 м нулевого с падением 13В в каждом проводнике). Поэтому при допустимом падении напряжения 2% или 5В сечение кабеля должно быть не меньше, чем 66 мм², или ближайшее большее стандартное значение – 75 мм².

Если расчет сечения по нагреву производится по рабочему току электродвигателя и на участке от вводного автомата до устройства, то расчёт по потерям необходимо производить по пусковому току с учётом всей длины кабелей: от магистрали до электромашины.

Сопротивление медного провода – это величина, влияющая на выбор кабелей и проводов для намотки катушек при проектировании электросхем, а также электродвигателей и трансформаторов. Знание того, как выполняется расчет сопротивления проводника, и необходимых формул поможет правильно спроектировать электропроводку и избежать аварийных ситуаций.

Видео

Электрическое сопротивление
физическая величина, которая показывает, какое препятствие создается току при его прохождении по проводнику
. Единицами измерения служат Омы, в честь Георга Ома. В своем законе он вывел формулу для нахождения сопротивления, которая приведена ниже.

Рассмотрим сопротивление проводников на примере металлов. Металлы имеют внутреннее строение в виде кристаллической решетки. Эта решетка имеет строгую упорядоченность, а её узлами являются положительно заряженные ионы. Носителями заряда в металле выступают “свободные” электроны, которые не принадлежат определенному атому, а хаотично перемещаются между узлами решетки. Из квантовой физики известно, что движение электронов в металле это распространение электромагнитной волны в твердом теле. То есть электрон в проводнике движется со скоростью света (практически), и доказано, что он проявляет свойства не только как частица, но еще и как волна. А сопротивление металла возникает в результате рассеяния электромагнитных волн (то есть электронов) на тепловых колебаниях решетки и её дефектах. При столкновении электронов с узлами кристаллической решетки часть энергии передается узлам, вследствие чего выделяется энергия. Эту энергию можно вычислить при постоянном токе , благодаря закону Джоуля-Ленца – Q=I 2 Rt. Как видите чем больше сопротивление, тем больше энергии выделяется.

Удельное сопротивление

Существует такое важное понятие как удельное сопротивление, это тоже самое сопротивление, только в единице длины. У каждого металла оно свое, например у меди оно равно 0,0175 Ом*мм2/м, у алюминия 0,0271 Ом*мм2/м. Это значит, брусок из меди длиной 1 м и площадью поперечного сечения 1 мм2 будет иметь сопротивление 0,0175 Ом, а такой же брусок, но из алюминия будет иметь сопротивление 0,0271 Ом. Выходит что электропроводность меди выше чем у алюминия. У каждого металла удельное сопротивление свое, а рассчитать сопротивление всего проводника можно по формуле

где p
– удельное сопротивление металла, l – длина проводника, s – площадь поперечного сечения.

Значения удельных сопротивлений приведены в таблице удельных сопротивлений металлов
(20°C)

Вещество

p
, Ом*мм 2 /2

α,10 -3 1/K

Алюминий

0.0271

Вольфрам

0.055

Железо

0.098

Золото

0.023

Латунь

0.025-0.06

Манганин

0.42-0.48

0,002-0,05

Медь

0. 0175

Никель

Константан

0.44-0.52

0.02

Нихром

0.15

Серебро

0.016

Цинк

0.059

Кроме удельного сопротивления в таблице есть значения ТКС, об этом коэффициенте чуть позже.

Зависимость удельного сопротивления от деформаций

При холодной обработке металлов давлением, металл испытывает пластическую деформацию. При пластической деформации кристаллическая решетка искажается, количество дефектов становится больше. С увеличением дефектов кристаллической решетки, сопротивление течению электронов по проводнику растет, следовательно, удельное сопротивление металла увеличивается. К примеру, проволоку изготавливают методом протяжки, это значит, что металл испытывает пластическую деформацию, в результате чего, удельное сопротивление растет. На практике для уменьшения сопротивления применяют рекристаллизационный отжиг, это сложный технологический процесс, после которого кристаллическая решетка как бы, “расправляется” и количество дефектов уменьшается, следовательно, и сопротивление металла тоже.

При растяжении или сжатии, металл испытывает упругую деформацию. При упругой деформации вызванной растяжением, амплитуды тепловых колебаний узлов кристаллической решетки увеличиваются, следовательно, электроны испытывают большие затруднения, и в связи с этим, увеличивается удельное сопротивление. При упругой деформации вызванной сжатием, амплитуды тепловых колебаний узлов уменьшаются, следовательно, электронам проще двигаться, и удельное сопротивление уменьшается.

Влияние температуры на удельное сопротивление

Как мы уже выяснили выше, причиной сопротивления в металле являются узлы кристаллической решетки и их колебания. Так вот, при увеличении температуры, тепловые колебания узлов увеличиваются, а значит, удельное сопротивление также увеличивается. Существует такая величина как температурный коэффициент сопротивления
(ТКС), который показывает насколько увеличивается, или уменьшается удельное сопротивление металла при нагреве или охлаждении. Например, температурный коэффициент меди при 20 градусах по цельсию равен 4.1
· 10 − 3 1/градус. Это означает что при нагреве, к примеру, медной проволоки на 1 градус цельсия, её удельное сопротивление увеличится на 4.1
· 10 − 3 Ом. Удельное сопротивление при изменении температуры можно вычислить по формуле

где r это удельное сопротивление после нагрева, r 0 – удельное сопротивление до нагрева, a – температурный коэффициент сопротивления, t 2 – температура до нагрева, t 1 — температура после нагрева.

Подставив наши значения, мы получим: r=0,0175*(1+0.0041*(154-20))=0,0271 Ом*мм 2 /м. Как видите наш брусок из меди длиной 1 м и площадью поперечного сечения 1 мм 2 , после нагрева до 154 градусов, имел бы сопротивление, как у такого же бруска, только из алюминия и при температуре равной 20 градусов цельсия.

Свойство изменения сопротивления при изменении температуры, используется в термометрах сопротивления. Эти приборы могут измерять температуру основываясь на показаниях сопротивления. У термометров сопротивления высокая точность измерений, но малые диапазоны температур.

На практике, свойства проводников препятствовать прохождению
тока используются очень широко. Примером может служить лампа накаливания, где нить из вольфрама, нагревается за счет высокого сопротивления металла, большой длины и узкого сечения. Или любой нагревательный прибор, где спираль разогревается благодаря высокому сопротивлению. В электротехнике, элемент главным свойством которого является сопротивление, называется – резистор . Резистор применяется практически в любой электрической схеме.

Понятие об электрическом сопротивлении и проводимости

Любое тело, по которому протекает электрический ток, оказывает ему определенное сопротивление.
Свойство материала проводника препятствовать прохождению через него электрического тока называется электрическим сопротивлением.

Электронная теория так объясняет сущность электрического сопротивления металлических проводников. Свободные электроны при движении по проводнику бесчисленное количество раз встречают на своем пути атомы и другие электроны и, взаимодействуя с ними, неизбежно теряют часть своей энергии. Электроны испытывают как бы сопротивление своему движению. Различные металлические проводники, имеющие различное атомное строение, оказывают различное сопротивление электрическому току.

Точно тем же объясняется сопротивление жидких проводников и газов прохождению электрического тока. Однако не следует забывать, что в этих веществах не электроны, а заряженные частицы молекул встречают сопротивление при своем движении.

Сопротивление обозначается латинскими буквами R или r
.

За единицу электрического сопротивления принят ом.

Ом есть сопротивление столба ртути высотой 106,3 см с поперечным сечением 1 мм2 при температуре 0° С.

Если, например, электрическое сопротивление проводника составляет 4 ом, то записывается это так: R = 4 ом или r
= 4ом.

Для измерения сопротивлений большой величины принята единица, называемая мегомом.

Один мегом равен одному миллиону ом.

Чем больше сопротивление проводника, тем хуже он проводит электрический ток, и, наоборот, чем меньше сопротивление проводника, тем легче электрическому току пройти через этот проводник.

Следовательно, для характеристики проводника (с точки зрения прохождения через него электрического тока) можно рассматривать не только его сопротивление, но и величину, обратную сопротивлению и называемую, проводимостью.

Электрической проводимостью
называется способность материала пропускать через себя электрический ток.

Так как проводимость есть величина, обратная сопротивлению, то и выражается она как 1/R
,обозначается проводимость латинской буквой g.

Влияние материала проводника, его размеров и окружающей температуры на величину электрического сопротивления

Сопротивление различных проводников зависит от материала, из которого они изготовлены. Для характеристики электрического сопротивления различных материалов введено понятие так называемого удельного сопротивления.

Удельным сопротивлением
называется сопротивление проводника длиной 1 м и площадью поперечного сечения 1 мм2. Удельное сопротивление обозначается буквой греческого алфавита р. Каждый материал, из которого изготовляется проводник, обладает своим удельным сопротивлением.

Например, удельное сопротивление меди равно 0,017, т. е. медный проводник длиной 1 м и сечением 1 мм2 обладает сопротивлением 0,017 ом. Удельное сопротивление алюминия равно 0,03, удельное сопротивление железа — 0,12, удельное сопротивление константана — 0,48, удельное сопротивление нихрома — 1-1,1.

Сопротивление проводника прямо пропорционально его длине, т. е. чем длиннее проводник, тем больше его электрическое сопротивление.

Сопротивление проводника обратно пропорционально площади его поперечного сечения, т. е. чем толще проводник, тем его сопротивление меньше, и, наоборот, чем тоньше проводник, тем его сопротивление больше.

Чтобы лучше понять эту зависимость, представьте себе две пары сообщающихся сосудов, причем у одной пары сосудов соединяющая трубка тонкая, а у другой — толстая. Ясно, что при заполнении водой одного из сосудов (каждой пары) переход ее в другой сосуд по толстой трубке произойдет гораздо быстрее, чем по тонкой, т. е. толстая трубка окажет меньшее сопротивление течению воды. Точно так же и электрическому току легче пройти по толстому проводнику, чем по тонкому, т. е. первый оказывает ему меньшее сопротивление, чем второй.

Электрическое сопротивление проводника равно удельному сопротивлению материала, из которого этот проводник сделан, умноженному на длину проводника и деленному на площадь площадь поперечного сечения проводника
:

R =
p l / S
,

Где — R —
сопротивление проводника, ом, l —
длина в проводника в м, S —
площадь поперечного сечения проводника, мм 2 .

Площадь поперечного сечения круглого проводника
вычисляется по формуле:

S =
Пи х
d 2 / 4

Где
Пи
— постоянная величина, равная 3,14; d — диаметр проводника.

А так определяется длина проводника:

l = S R / p
,

Эта формула дает возможность определить длину проводника, его сечение и удельное сопротивление, если известны остальные величины, входящие в формулу.

Если же необходимо определить площадь поперечного сечения проводника, то формулу приводят к следующему виду:

S =
p l / R

Преобразуя ту же формулу и решив равенство относительно р, найдем удельное сопротивление проводника:

р =
R S / l

Последней формулой приходится пользоваться в тех случаях, когда известны сопротивление и размеры проводника, а его материал неизвестен и к тому же трудно определим по внешнему виду. Для этого надо определить удельное сопротивление проводника и, пользуясь таблицей, найти материал, обладающий таким удельным сопротивлением.

Еще одной причиной, влияющей на сопротивление проводников, является температура
.

Установлено, что с повышением температуры сопротивление металлических проводников возрастает, а с понижением уменьшается. Это увеличение или уменьшение сопротивления для проводников из чистых металлов почти одинаково и в среднем равно 0,4% на 1°C
. Сопротивление жидких проводников и угля с увеличением температуры уменьшается.

Электронная теория строения вещества дает следующее объяснение увеличению сопротивления металлических проводников с повышением температуры. При нагревании проводник получает тепловую энергию, которая неизбежно передается всем атомам вещества, в результате чего возрастает интенсивность их движения. Возросшее движение атомов создает большее сопротивление направленному движению свободных электронов, отчего и возрастает сопротивление проводника. С понижением же температуры создаются лучшие условия для направленного движения электронов, и сопротивление проводника уменьшается. Этим объясняется интересное явление — сверхпроводимость металлов
.

Сверхпроводимость
, т. е. уменьшение сопротивления металлов до нуля, наступает при огромной отрицательной температуре —
273° C
, называемой абсолютным нулем. При температуре абсолютного нуля атомы металла как бы застывают на месте, совершенно не препятствуя движению электронов.

Зависимость сопротивления от температуры

«Всё
это так не потому что я такой умный.

Это
всё из-за того, что я долго

не
сдаюсь при решении задач»

Альберт
Эйнштейн

Данная
тема посвящена решению задач на зависимость сопротивления проводника от
температуры

Задача
1.

Найдите сопротивление алюминиевого провода длиной 20 м и площадью
поперечного сечения 2 мм2 при температуре 70 ºС,
учитывая то, что в таблице указаны значения удельных сопротивлений при
температуре 20 ºС.



ДАНО:

РЕШЕНИЕ

Зависимость удельного сопротивления
от температуры имеет вид

Тогда при температуре
70 ºС

Сопротивление проводника
можно определить по формуле

Тогда при температуре
70 ºС

Ответ:
0,32 Ом.

Задача
2.

На баллоне лампы накаливания написано 220 В, 100 Вт. Когда нить накала была
холодной, т.е. комнатной температуры, на неё подали напряжение 2 В и
измерили силу тока. Ток оказался равен 50 мА. Найдите приблизительно
температуру накала, нити, учитывая то, что она сделана из вольфрама.



ДАНО:

СИ

РЕШЕНИЕ

Из формулы для определения мощности электрического тока
определим сопротивление

Запишем закон Ома для участка цепи

Тогда

Запишем зависимость сопротивления от температуры

Запишем выражение для
сопротивления при некоторой температуре t1

Тогда отношение сопротивлений

Выразим из данной формулы температуру t

Значения сопротивлений при температурах t и t1 равны

Тогда

Ответ:
приблизительная
температура накала нити 2462 ºС.

Задача
3.

Медный провод нагревается под действием электрического тока от 0 до 25 ºС
за 3 мин. Через провод протекает ток 50 А. Предполагая, что изменение силы
тока незначительно, найдите работу тока при нагревании провода. Сопротивление
провода при 0 ºС равно 200 мОм.



ДАНО:

СИ

РЕШЕНИЕ

Работа электрического тока рассчитывается по формуле

Мощность электрического тока

Начальное сопротивление –
это сопротивление при нуле градусах

Чтобы вычислить
сопротивление при 25 ºС, необходимо записать зависимость
сопротивления от температуры

Вычислим мощность тока при 0
и 25 ºС

Как видно из формулы,
мощность линейно зависит от сопротивления, а сопротивление, в свою очередь,
линейно зависит от температуры. Поэтому, мощность будет линейно зависеть от
температуры.

Чтобы найти работу тока,
необходимо построить график зависимости мощности от времени.

Чтобы найти работу тока, необходимо
найти площадь под графиком. Площадь трапеции равна

Тогда работа

Ответ:
94,5 кДж.

Задача
4.

К концам проволоки приложено некоторое напряжение. По мере нагревания проволоки
до 50 ºС, сила тока уменьшилась от 1 до 0,9 А. Найдите начальную
температуру проволоки, если её температурный коэффициент сопротивления равен 0,004 ºС–1.



ДАНО:

РЕШЕНИЕ

Запишем закон Ома для участка цепи

Исходя из данного закона запишем сопротивление проволоки
при начальной и конечной температурах

Отношение этих сопротивлений равно

Зависимость сопротивления от температуры

Тогда для начальной и конечной температуры сопротивления
равня

Отношения этих сопротивлений

Приравняем две формулы выражающие отношения сопротивлений

Из последней формулы выразим начальную температуру

Ответ:
20 ºС

Задача
5.

Две одинаковые проволоки подключены параллельно. Одна из этих проволок помещена
в тающий лёд, а другая находится при температуре 20 ºС. Температурный
коэффициент сопротивления проволок равен 0,01 ºС–1.
Сравните общее сопротивление этого участка с сопротивлением, которое было бы,
если бы обе проволоки находились при температуре 20 ºС.



ДАНО:

РЕШЕНИЕ

Зависимость сопротивления от температуры имеет вид

Тогда при температурах 0 ºС и 20 ºС

При параллельном соединении

Если две одинаковые проволоки находятся при одной и той же
температуре, то их сопротивления равны

При параллельном соединении

Тогда отношение сопротивлений равно

Ответ:
если бы две проволоки находились при температуре 20 ºС, то
сопротивление данного участка было бы в 1,1 раз больше.

Расчет сопротивления проводника. Удельное сопротивление

1. Расчет сопротивления проводника. Удельное сопротивление

Горбунова В.А учитель физики
МБОУ Черемуховская СОШ
Новошешминского района
2013 год
Задачи урока:
обучения: установить зависимость сопротивления
проводника от его длины, площади поперечного сечения
и вещества, из которого он изготовлен.
воспитания: воспитание мировоззренческих понятий;
познаваемость окружающего мира; этики работы в
парах.
развития: развивать элементы творческого поиска на
основе приема обобщения знаний, умение
анализировать, наблюдать, собирать электрические
цепи, чертить схемы, развивать навыки практической
работы, интерес к предмету путём выполнения разных
заданий.
измерять.
Наука начинается с тех пор, как начинают
Точная наука немыслима без меры.
Д.И.Менделеев
Цель урока: получить соотношение между сопротивлением
проводника, его длиной, площадью поперечного сечения и
удельным сопротивлением.
Оборудование: источник тока, амперметр, вольтметр, линейка,
ключ, исследуемые проводники, соединительные провода,
компьютер, проектор.

4. Определение силы тока:

Сила тока – физическая величина,
равная отношению заряда,
прошедшего через поперечное
сечение проводника, ко времени его
прохождения.

5. Андре-Мари Ампер

(1775 — 1836)
французский физик и математик
Условное обозначение
в электрической схеме
Правила
подключения:
Снимите показания с
приборов
I=1,4 A
I=3 A

9. Напряжение

Электрическое напряжение – физическая величина,
характеризующая электрическое поле.
A
U
q
Алессандро Вольта –
Итальянский физик и химик.

10. Напряжение

Прибор для измерения
электрического напряжения вольтметр
На схемах вольтметр изображают
кружком с буквой V внутри.

11. Напряжение

В
цепь вольтметр, в отличие от
амперметра, включается параллельно.
сопротивление
К=П
И
электричество А=Е
С=К
«Когда я первый раз прочел
теорию Ома, она мне показалась
молнией, вдруг осветившей
комнату, погруженную во мрак»
Дж. Генри
Золотая медаль Лондонского Королевского общества –
награда Георга Ома

17. Применяя закон Ома для участка цепи, заполните таблицу. Заполни таблицу

I
0,2 А
U
4 В
R
4
8
20
А
В
Ом
0,22 А
6,6 В
55
Ом
I, А
6
Какой из проводников имеет наибольшее
сопротивление?
А
4
В
2
Какой из проводников имеет наименьшее
сопротивление?
С
0
2
4
6
8
10
U, В

19. Опытным путем Георг Ом установил, от каких факторов зависит сопротивление проводника

20. Зависимость сопротивления проводника от его длины

S1=S2=S
никелин
l
R
2l
2R
Таким образом, сопротивление проводника зависит
прямопропорционально от его длины:
R~l

21.

Зависимость сопротивления проводника от площади его поперечного сечения

l1=l2=l
никелин
S
2S
R
R/2
Таким образом, сопротивление проводника зависит
обратнопропорционально от площади его поперечного
сечения:
R ~ 1/S

22. Зависимость сопротивления проводника от рода материала

l, S, никелин
R1
l, S, нихром

R2
Очевидно, что сопротивление проводника зависит от рода
вещества, из которого изготовлен проводник

23. Выводы

Сопротивление зависит от длины
проводника, чем больше длина
проводника тем больше его
сопротивление.
Сопротивление проводника зависит от
площади поперечного сечения: чем
меньше площадь сечения проводника,
тем больше сопротивление.
Сопротивление проводника зависит от
рода вещества (материала), из которого
он изготовлен.
Зависимость сопротивления от
геометрических размеров проводника
(длины и площади поперечного
сечения) и вещества, из которого он
изготовлен, впервые установил Георг
Ом.
R
L
S
;
Это выражение позволяет вычислять длину проводника,
поперечное сечение и удельное сопротивление
проводника.
RS
;
L
L
RS
;
S
L
R
.

25. Удельное сопротивление проводника, ρ —

Удельное сопротивление
проводника, ρ это физическая величина, показывающая, каково сопротивление
проводника из данного вещества длиной 1 м и площадью
поперечного сечения 1м2
l
R ,
S
RS
RS l,
l
Ом мм 2
м
Свинец, Pb: ρ=0,21 Ом·мм2/м – это значит, что
сопротивление свинцового проводника длиной
1 м и площадью поперечного сечения 1 мм2 равно 0,21
Ом.
Стр. 106, пример 1.
Прочитай и запиши в тетрадь
решение задачи, приведенной
на странице 106.

28. Решение задач

1. Сколько метров никелиновой проволоки сечением
0,1 мм2 потребуется для изготовления реостата с
сопротивлением 180 Ом?
l-?
ρ=0,4 Ом·мм2/м
S=0,1 мм2
R=180 Ом
l
R , RS l ,
S
l
RS
Ом мм 2 м
l
м
2
Ом мм
180 0,1
l
45 ( м)
0,4
При устройстве молниеотвода
использовали железный провод
сечением 50 мм2 и длиной 25 м.
Определите его сопротивление.
Ответ: 0,05 Ом

30. Задача

Задача. Определите сопротивление телеграфного провода между
Южно-Сахалинском и Томари, если расстояние между городами 180
км, а провода сделаны из железной проволоки площадью поперечного
сечения 12 мм2
Задача. Рассчитайте сопротивление медного контактного провода,
подвешенного для питания трамвайного двигателя, если длина
провода равна 5 км, а площадь поперечного сечения — 0,65 см2 .
Задача. Какой длины надо взять медную проволоку площадью
поперечного сечения 0,5 мм2 , чтобы сопротивление ее было равно 34
Ом?
Задача. Вычислите, каким сопротивлением обладает нихромовый
проводник длиной 5 м и площадью поперечного сечения 0,75 мм2 .
Электрическое сопротивление
R, [R]=1 Ом , 1 Ом = 1В/1А;
L
R=ρ __
S
ρ – удельное
сопротивление
______
Ом
мм2
[ρ] =
м

32. Выводы

Сопротивление зависит от длины
проводника, чем больше длина
проводника тем больше его
сопротивление.
Сопротивление проводника зависит от
площади поперечного сечения: чем
меньше площадь сечения проводника,
тем больше сопротивление.
Сопротивление проводника зависит от
рода вещества (материала), из которого
он изготовлен.

33. Домашнее задание

§45, 46, упражнение 20, № 2 (а), 4.

Сопротивление проводника зависит от его длины. Электрическое сопротивление проводника

Содержание:


При проектировании электрических сетей в квартирах или частных домах в обязательном порядке выполняется расчет сечения проводов и кабелей. Для проведения вычислений используются такие показатели, как значение потребляемой мощности и сила тока, которая будет проходить по сети. Сопротивление не принимается в расчет из-за малой протяженности кабельных линий. Однако этот показатель необходим при большой длине ЛЭП и перепадах напряжения на различных участках. Особое значение имеет сопротивление медного провода. Такие провода все чаще используются в современных сетях, поэтому их физические свойства должны обязательно учитываться при проектировании.

Понятия и значение сопротивления

Электрическое сопротивление материалов широко используется и учитывается в электротехнике. Данная величина позволяет установить основные параметры проводов и кабелей, особенно при скрытом способе их прокладки. В первую очередь устанавливается точная длина проложенной линии и материал, использованный для производства провода. Вычислив первоначальные данные, вполне возможно измеряемого кабеля.

По сравнению с обычной электрической проводкой, в электронике параметрам сопротивления придается решающее значение. Оно рассматривается и сопоставляется в совокупности с другими показателями, присутствующими в электронных схемах. В этих случаях неправильно подобранное сопротивление провода, может вызвать сбой в работе всех элементов системы. Такое может произойти, если для подключения к блоку питания компьютера воспользоваться слишком тонким проводом. Произойдет незначительное снижение напряжения в проводнике, что вызовет некорректную работу компьютера.

Сопротивление в медном проводе зависит от многих факторов, и в первую очередь от физических свойств самого материала. Кроме того, учитывается диаметр или сечение проводника, определяемые по формуле или специальной таблице.

Таблица

На сопротивление медного проводника оказывают влияние несколько дополнительных физических величин. Прежде всего должна учитываться температура окружающей среды. Всем известно, что при повышении температуры проводника, наблюдается рост его сопротивления. Одновременно с этим происходит снижение силы тока из-за обратно пропорциональной зависимости обеих величин. В первую очередь это касается металлов с положительным температурным коэффициентом. Примером отрицательного коэффициента является вольфрамовый сплав, применяющийся в лампах накаливания. В этом сплаве сила тока не снижается даже при очень высоком нагреве.

Как рассчитать сопротивление

Для расчетов сопротивления медного провода существует несколько способов. К наиболее простым относится табличный вариант, где указаны взаимосвязанные параметры. Поэтому, кроме сопротивления, определяется сила тока, диаметр или сечение провода.

Во втором случае используются разнообразные . В каждый из них вставляется набор физических величин медного провода, с помощью которых получаются точные результаты. В большинстве подобных калькуляторов используется в размере 0,0172 Ом*мм 2 /м. В некоторых случаях такое усредненное значение может повлиять на точность вычислений.

Наиболее сложным вариантом считаются ручные вычисления, с использованием формулы: R = p x L/S, в которой р — удельное сопротивление меди, L — длина проводника и S — сечение этого проводника. Следует отметить, что сопротивление медного провода таблица определяет, как одно из наиболее низких. Более низким значением обладает лишь серебро.


Когда производится расчет сечения кабеля, то в частном домостроении или в квартирах для определения этой величины используются два показателя: потребляемая мощность сети и сила тока, проходящая по разводке. Сопротивление в данном случае роли не играет. Все дело в небольшой длине проводов. А вот если длина линии электропередач достаточно большая, то без определения данного показателя здесь не обойтись. К примеру, на начале участка напряжение будет 220-2240 вольт, а на конце уже заниженное 200-220 вольт. А так как все чаще в проводке используются медные кабели и провода, то наша задача в этой статье рассмотреть сопротивление медного провода (таблица сопротивления проводов будет ниже приложена).

Что нам дает сопротивление в общем? В принципе, с его помощью можно узнать параметры используемого провода или материал, из которого он изготовлен. К примеру, если для прокладки линии электропередачи использовался скрытый способ, то зная сопротивление линии, можно точно сказать, какой она длины. Ведь часто прокладка производится под землей и непрямолинейным способом. Или еще один вариант, зная длину участка и его сопротивление можно подсчитать диаметр используемого кабеля, а через него и его сечение. Плюс, зная данную величину, можно узнать материал, из которого этот провод был изготовлен. Это все говорит о том, что не стоит сбрасывать со счетов данный показатель.

Все это касалось электрической проводки, но когда дело касается электроники, то в этой области без определения сопротивления и сопоставления его с другими параметрами не обойтись. В некоторых случаях данный параметр может сыграть решающую роль, даже неправильный подбор провода по сопротивлению может привести к тому, что подключаемый к такому проводнику прибор просто не будет работать. К примеру, если к блоку питания обычного компьютера подключить очень тонкий провод. Напряжение в таком проводнике станет низким, не намного, но этого будет хватать, чтобы компьютер работал некорректно.

От чего зависит сопротивление

Так как мы говорим о медном проводе, то первое от чего зависит этот физический параметр, это медь, то есть, сырьевой материал. Второе – это размеры проводника, а, точнее, его диаметр или сечение (обе величины связаны между собой формулой).

Конечно, есть дополнительные физические величины, которые влияют на сопротивление проводника. К примеру, температура окружающей среды. Ведь известно, что при повышении температуры самого провода, его сопротивление увеличивается. А так как этот показатель находится в обратной зависимости от силы (плотность) тока, соответственно ток при повышении сопротивления, наоборот, снижается. Правда, это относится к тем металлам, которые являются обладателями положительного температурного коэффициента. Для примера можно привести сплав вольфрама, который используется для нити накала лампочки. Такому материалу изменения силы (плотность) тока не страшны при высоком нагреве, потому что этот металл обладает отрицательным температурным коэффициентом.

Расчет сопротивления

Сегодня все сделано для человека. И даже такой простой расчет можно сделать несколькими способами. Есть простые, есть сложные. Начнем с простых.

Первый вариант табличный. В чем его простота? К примеру, таблица на нижнем рисунке.

Здесь все четко показано и взаимосвязано. Зная определенные размеры медного провода, можно определить его сопротивление и силу тока, которую провод может выдержать. Или, наоборот, имея в наличие показатели сопротивления или силы (плотность) тока, которые, кстати, можно определить мультиметром, можно легко определить сечение или диаметр проводника. Данный вариант самый удобный, таблицы можно найти в свободном доступе в интернете.

Второй способ определения – с помощью калькулятора (онлайн). Таких интернетовских приспособлений великое множество, работать с ними удобно и легко. Можно в такой калькулятор вставлять физические величины медного проводника и получать размерные показатели, или, наоборот. Правда, основная масса таких калькуляторов в своей программе имеет одно стандартное значение – это удельное сопротивление меди, равное 0,0172 Ом·мм²/м.

И самый сложный вариант расчета – это провести его своими руками, используя формулу. Вот она: R=pl/S, где:

  • р – это то самое удельное сопротивление меди;
  • l – длина медного провода;
  • S – его сечение.

Хотелось бы отметить, что медь обладает одним из самых низких удельных сопротивлений. Ниже него только серебро – 0,016.

Определить сечение проводника можно через формулу, где основным параметром является его диаметр. А вот определить диаметр можно разными способами, кстати, такая статья на нашем сайте есть, можете прочитать и получить полную и достоверную информацию.

Заключение о теме

Подводим итог всему вышесказанному. Конечно, никто не будет учитывать сопротивление электрической разводки медным кабелем в доме или квартире. Но если дело касается прокладки воздушных или подземных линий электропередач, к примеру, от подстанции до дачного участка, то данный показатель придется учитывать обязательно. Ведь именно он повлияет на качество напряжения в сети дома. А вот рассчитать параметры укладываемых кабелей можно будет разными способами, где показатель сопротивления медного провода (таблица приложена) является одним из основных.

Понятие об электрическом сопротивлении и проводимости

Любое тело, по которому протекает электрический ток, оказывает ему определенное сопротивление.
Свойство материала проводника препятствовать прохождению через него электрического тока называется электрическим сопротивлением.

Электронная теория так объясняет сущность электрического сопротивления металлических проводников. Свободные электроны при движении по проводнику бесчисленное количество раз встречают на своем пути атомы и другие электроны и, взаимодействуя с ними, неизбежно теряют часть своей энергии. Электроны испытывают как бы сопротивление своему движению. Различные металлические проводники, имеющие различное атомное строение, оказывают различное сопротивление электрическому току.

Точно тем же объясняется сопротивление жидких проводников и газов прохождению электрического тока. Однако не следует забывать, что в этих веществах не электроны, а заряженные частицы молекул встречают сопротивление при своем движении.

Сопротивление обозначается латинскими буквами R или r
.

За единицу электрического сопротивления принят ом.

Ом есть сопротивление столба ртути высотой 106,3 см с поперечным сечением 1 мм2 при температуре 0° С.

Если, например, электрическое сопротивление проводника составляет 4 ом, то записывается это так: R = 4 ом или r
= 4ом.

Для измерения сопротивлений большой величины принята единица, называемая мегомом.

Один мегом равен одному миллиону ом.

Чем больше сопротивление проводника, тем хуже он проводит электрический ток, и, наоборот, чем меньше сопротивление проводника, тем легче электрическому току пройти через этот проводник.

Следовательно, для характеристики проводника (с точки зрения прохождения через него электрического тока) можно рассматривать не только его сопротивление, но и величину, обратную сопротивлению и называемую, проводимостью.

Электрической проводимостью
называется способность материала пропускать через себя электрический ток.

Так как проводимость есть величина, обратная сопротивлению, то и выражается она как 1/R
,обозначается проводимость латинской буквой g.

Влияние материала проводника, его размеров и окружающей температуры на величину электрического сопротивления

Сопротивление различных проводников зависит от материала, из которого они изготовлены. Для характеристики электрического сопротивления различных материалов введено понятие так называемого удельного сопротивления.

Удельным сопротивлением
называется сопротивление проводника длиной 1 м и площадью поперечного сечения 1 мм2. Удельное сопротивление обозначается буквой греческого алфавита р. Каждый материал, из которого изготовляется проводник, обладает своим удельным сопротивлением.

Например, удельное сопротивление меди равно 0,017, т. е. медный проводник длиной 1 м и сечением 1 мм2 обладает сопротивлением 0,017 ом. Удельное сопротивление алюминия равно 0,03, удельное сопротивление железа — 0,12, удельное сопротивление константана — 0,48, удельное сопротивление нихрома — 1-1,1.

Сопротивление проводника прямо пропорционально его длине, т. е. чем длиннее проводник, тем больше его электрическое сопротивление.

Сопротивление проводника обратно пропорционально площади его поперечного сечения, т. е. чем толще проводник, тем его сопротивление меньше, и, наоборот, чем тоньше проводник, тем его сопротивление больше.

Чтобы лучше понять эту зависимость, представьте себе две пары сообщающихся сосудов, причем у одной пары сосудов соединяющая трубка тонкая, а у другой — толстая. Ясно, что при заполнении водой одного из сосудов (каждой пары) переход ее в другой сосуд по толстой трубке произойдет гораздо быстрее, чем по тонкой, т. е. толстая трубка окажет меньшее сопротивление течению воды. Точно так же и электрическому току легче пройти по толстому проводнику, чем по тонкому, т. е. первый оказывает ему меньшее сопротивление, чем второй.

Электрическое сопротивление проводника равно удельному сопротивлению материала, из которого этот проводник сделан, умноженному на длину проводника и деленному на площадь площадь поперечного сечения проводника
:

R =
p l / S
,

Где — R —
сопротивление проводника, ом, l —
длина в проводника в м, S —
площадь поперечного сечения проводника, мм 2 .

Площадь поперечного сечения круглого проводника
вычисляется по формуле:

S =
Пи х
d 2 / 4

Где
Пи
— постоянная величина, равная 3,14; d — диаметр проводника.

А так определяется длина проводника:

l = S R / p
,

Эта формула дает возможность определить длину проводника, его сечение и удельное сопротивление, если известны остальные величины, входящие в формулу.

Если же необходимо определить площадь поперечного сечения проводника, то формулу приводят к следующему виду:

S =
p l / R

Преобразуя ту же формулу и решив равенство относительно р, найдем удельное сопротивление проводника:

р =
R S / l

Последней формулой приходится пользоваться в тех случаях, когда известны сопротивление и размеры проводника, а его материал неизвестен и к тому же трудно определим по внешнему виду. Для этого надо определить удельное сопротивление проводника и, пользуясь таблицей, найти материал, обладающий таким удельным сопротивлением.

Еще одной причиной, влияющей на сопротивление проводников, является температура
.

Установлено, что с повышением температуры сопротивление металлических проводников возрастает, а с понижением уменьшается. Это увеличение или уменьшение сопротивления для проводников из чистых металлов почти одинаково и в среднем равно 0,4% на 1°C
. Сопротивление жидких проводников и угля с увеличением температуры уменьшается.

Электронная теория строения вещества дает следующее объяснение увеличению сопротивления металлических проводников с повышением температуры. При нагревании проводник получает тепловую энергию, которая неизбежно передается всем атомам вещества, в результате чего возрастает интенсивность их движения. Возросшее движение атомов создает большее сопротивление направленному движению свободных электронов, отчего и возрастает сопротивление проводника. С понижением же температуры создаются лучшие условия для направленного движения электронов, и сопротивление проводника уменьшается. Этим объясняется интересное явление — сверхпроводимость металлов
.

Сверхпроводимость
, т. е. уменьшение сопротивления металлов до нуля, наступает при огромной отрицательной температуре —
273° C
, называемой абсолютным нулем. При температуре абсолютного нуля атомы металла как бы застывают на месте, совершенно не препятствуя движению электронов.

Электрическое сопротивление
физическая величина, которая показывает, какое препятствие создается току при его прохождении по проводнику
. Единицами измерения служат Омы, в честь Георга Ома. В своем законе он вывел формулу для нахождения сопротивления, которая приведена ниже.

Рассмотрим сопротивление проводников на примере металлов. Металлы имеют внутреннее строение в виде кристаллической решетки. Эта решетка имеет строгую упорядоченность, а её узлами являются положительно заряженные ионы. Носителями заряда в металле выступают “свободные” электроны, которые не принадлежат определенному атому, а хаотично перемещаются между узлами решетки. Из квантовой физики известно, что движение электронов в металле это распространение электромагнитной волны в твердом теле. То есть электрон в проводнике движется со скоростью света (практически), и доказано, что он проявляет свойства не только как частица, но еще и как волна. А сопротивление металла возникает в результате рассеяния электромагнитных волн (то есть электронов) на тепловых колебаниях решетки и её дефектах. При столкновении электронов с узлами кристаллической решетки часть энергии передается узлам, вследствие чего выделяется энергия. Эту энергию можно вычислить при постоянном токе , благодаря закону Джоуля-Ленца – Q=I 2 Rt. Как видите чем больше сопротивление, тем больше энергии выделяется.

Удельное сопротивление

Существует такое важное понятие как удельное сопротивление, это тоже самое сопротивление, только в единице длины. У каждого металла оно свое, например у меди оно равно 0,0175 Ом*мм2/м, у алюминия 0,0271 Ом*мм2/м. Это значит, брусок из меди длиной 1 м и площадью поперечного сечения 1 мм2 будет иметь сопротивление 0,0175 Ом, а такой же брусок, но из алюминия будет иметь сопротивление 0,0271 Ом. Выходит что электропроводность меди выше чем у алюминия. У каждого металла удельное сопротивление свое, а рассчитать сопротивление всего проводника можно по формуле

где p
– удельное сопротивление металла, l – длина проводника, s – площадь поперечного сечения.

Значения удельных сопротивлений приведены в таблице удельных сопротивлений металлов
(20°C)

Вещество

p
, Ом*мм 2 /2

α,10 -3 1/K

Алюминий

0.0271

Вольфрам

0.055

Железо

0.098

Золото

0.023

Латунь

0.025-0.06

Манганин

0.42-0.48

0,002-0,05

Медь

0. 0175

Никель

Константан

0.44-0.52

0.02

Нихром

0.15

Серебро

0.016

Цинк

0.059

Кроме удельного сопротивления в таблице есть значения ТКС, об этом коэффициенте чуть позже.

Зависимость удельного сопротивления от деформаций

При холодной обработке металлов давлением, металл испытывает пластическую деформацию. При пластической деформации кристаллическая решетка искажается, количество дефектов становится больше. С увеличением дефектов кристаллической решетки, сопротивление течению электронов по проводнику растет, следовательно, удельное сопротивление металла увеличивается. К примеру, проволоку изготавливают методом протяжки, это значит, что металл испытывает пластическую деформацию, в результате чего, удельное сопротивление растет. На практике для уменьшения сопротивления применяют рекристаллизационный отжиг, это сложный технологический процесс, после которого кристаллическая решетка как бы, “расправляется” и количество дефектов уменьшается, следовательно, и сопротивление металла тоже.

При растяжении или сжатии, металл испытывает упругую деформацию. При упругой деформации вызванной растяжением, амплитуды тепловых колебаний узлов кристаллической решетки увеличиваются, следовательно, электроны испытывают большие затруднения, и в связи с этим, увеличивается удельное сопротивление. При упругой деформации вызванной сжатием, амплитуды тепловых колебаний узлов уменьшаются, следовательно, электронам проще двигаться, и удельное сопротивление уменьшается.

Влияние температуры на удельное сопротивление

Как мы уже выяснили выше, причиной сопротивления в металле являются узлы кристаллической решетки и их колебания. Так вот, при увеличении температуры, тепловые колебания узлов увеличиваются, а значит, удельное сопротивление также увеличивается. Существует такая величина как температурный коэффициент сопротивления
(ТКС), который показывает насколько увеличивается, или уменьшается удельное сопротивление металла при нагреве или охлаждении. Например, температурный коэффициент меди при 20 градусах по цельсию равен 4.1
· 10 − 3 1/градус. Это означает что при нагреве, к примеру, медной проволоки на 1 градус цельсия, её удельное сопротивление увеличится на 4.1
· 10 − 3 Ом. Удельное сопротивление при изменении температуры можно вычислить по формуле

где r это удельное сопротивление после нагрева, r 0 – удельное сопротивление до нагрева, a – температурный коэффициент сопротивления, t 2 – температура до нагрева, t 1 — температура после нагрева.

Подставив наши значения, мы получим: r=0,0175*(1+0.0041*(154-20))=0,0271 Ом*мм 2 /м. Как видите наш брусок из меди длиной 1 м и площадью поперечного сечения 1 мм 2 , после нагрева до 154 градусов, имел бы сопротивление, как у такого же бруска, только из алюминия и при температуре равной 20 градусов цельсия.

Свойство изменения сопротивления при изменении температуры, используется в термометрах сопротивления. Эти приборы могут измерять температуру основываясь на показаниях сопротивления. У термометров сопротивления высокая точность измерений, но малые диапазоны температур.

На практике, свойства проводников препятствовать прохождению
тока используются очень широко. Примером может служить лампа накаливания, где нить из вольфрама, нагревается за счет высокого сопротивления металла, большой длины и узкого сечения. Или любой нагревательный прибор, где спираль разогревается благодаря высокому сопротивлению. В электротехнике, элемент главным свойством которого является сопротивление, называется – резистор . Резистор применяется практически в любой электрической схеме.

В своей работе электрик часто сталкивается с вычислением различных величин и преобразований. Так для корректного подбора кабеля приходится подбирать нужное сечение. Логика выбора сечения основана на зависимости сопротивления от длины линии и площади сечения проводника. В этой статье мы рассмотрим, как выполняется расчет сопротивления провода по его геометрическим размерам.


Формула для расчета

Любые вычисления начинаются с формулы. Основной формулой для расчета сопротивления проводника является:

R=(ρ*l)/S

Где R – сопротивление в Омах, ρ – удельное сопротивление, l – длина в м, S – площадь поперечного сечения провода в мм 2 .

Эта формула подходит для расчета сопротивления провода по сечению и длине. Из неё следует, что в зависимости от длины изменяется сопротивление, чем длиннее – тем больше. И от площади сечения – наоборот, чем толще провод (большое сечение), тем меньше сопротивление. Однако непонятной остаётся величина, обозначенная буквой ρ (Ро).

Удельное сопротивление

Удельное сопротивление – это табличная величина, для каждого металла она своя. Она нужна для расчета и зависит от кристаллической решетки металла и структуры атомов.

Из таблицы видно, что самое меньшее сопротивление у серебра, для медного кабеля оно равняется 0,017 Ом*мм 2 /м. Такая размерность говорит нам, сколько приходится Ом при сечении в 1 миллиметр квадратный и длине в 1 метр.

Кстати, серебряное покрытие используется в контактах коммутационных аппаратов, автоматических выключателей, реле и прочего. Это снижает , повышает срок службы и уменьшает . При этом в контактах измерительной и точной аппаратуры используют позолоченные контакты из-за того, что они слабо окисляются или вообще не окисляются.

У алюминия, который часто использовался в электропроводке раньше, сопротивление в 1,8 раза больше чем у меди, равняется 2,82*10 -8 Ом*мм 2 /м. Чем больше сопротивление проводника, тем сильнее он греется. Поэтому при одинаковом сечении алюминиевый кабель может передать меньший ток, чем медный, это и стало основной причиной почему все современные электрики используют . У нихрома, который используется в нагревательных приборах оно в 100 раз больше чем у меди 1,1*10 -6 Ом*мм 2 /м.

Расчет по диаметру

На практике часто бывает так, что площадь поперечного сечения жилы не известна. Без этого значения ничего рассчитать не получится. Чтобы узнать её, нужно измерить диаметр. Если жила тонка, можно взять гвоздь или любой другой стержень, намотать на него 10 витков провода, обычной линейкой измерить длину получившейся спирали и разделить на 10, так вы узнаете диаметр.

Ну, или просто замерить штангенциркулем. Расчет сечения выполняется по формуле:

Обязательны ли расчеты?

Как мы уже сказали, сечение провода выбирают исходя из предполагаемого тока и сопротивления металла, из которого изготовлены жилы. Логика выбора заключается в следующем: сечение подбирают таким способом, чтобы сопротивление при заданной длине не приводило к значительным просадкам напряжения. Чтобы не проводить ряд расчетов, для коротких линий (до 10-20 метров) есть достаточно точные таблицы:

В этой таблице указаны типовые значения сечения медных и алюминиевых жил и номинальные токи через них. Для удобства указана мощность нагрузки, которую выдержит эта линия. {2} \rho \frac{d l}{S}(1)$$

называют сопротивлением участка цепи между сечениями 1 и 2. В выражении (1) имеем
$\rho$ – удельное сопротивление проводника,
S – площадь поперечного сечения проводника, dl — элемент длины проводника.

Если проводник является однородным ($\rho$=const) и имеет форму цилиндра (S=const), то формула (1) может быть представлена как:

$$R=\rho \frac{l}{S}(2)$$

где l – длина участка рассматриваемого проводника.

Надо отметить, что удельное сопротивление проводника ($\rho$) –
это сопротивление проводника единичной длины с поперечным сечением равным единице. Или иначе говорят, что удельное сопротивление
вещества – это сопротивление куба с ребром 1 м изготовленного из рассматриваемого вещества, которое выражено в Ом, при токе,
который параллелен ребру куба. Величина обратная удельному сопротивлению:

$$\sigma=\frac{1}{\rho}(3)$$

называется удельной проводимостью. Измеряется удельное сопротивление в системе СИ
в [$\rho$]=Ом•м. {-2}}=100$$

Ответ. n=100

Читать дальше: Формула внутренней энергии.

Учебное пособие по физике: электрическое сопротивление

Электрон, движущийся по проводам и нагрузкам внешней цепи, встречает сопротивление. Сопротивление препятствует прохождению заряда. Для электрона путешествие от терминала к терминалу не является прямым маршрутом. Скорее, это зигзагообразный путь, который возникает в результате бесчисленных столкновений с неподвижными атомами в проводящем материале. Электроны сталкиваются с сопротивлением — препятствием для их движения. В то время как разность электрических потенциалов, установленная между двумя выводами , способствует перемещению заряда , — это сопротивление, которое препятствует ему.Скорость, с которой заряд проходит от терминала к терминалу, является результатом совместного действия этих двух величин.

Переменные, влияющие на электрическое сопротивление

Поток заряда по проводам часто сравнивают с потоком воды по трубам. Сопротивление потоку заряда в электрической цепи аналогично эффектам трения между водой и поверхностями трубы, а также сопротивлению, создаваемому препятствиями, присутствующими на ее пути.Именно это сопротивление препятствует потоку воды и снижает как скорость потока, так и скорость сноса . Подобно сопротивлению потоку воды, общее сопротивление потоку заряда в проводе электрической цепи зависит от некоторых четко идентифицируемых переменных.

Во-первых, общая длина проводов влияет на величину сопротивления. Чем длиннее провод, тем большее сопротивление будет. Существует прямая зависимость между величиной сопротивления, с которым сталкивается заряд, и длиной провода, который он должен пройти.В конце концов, если сопротивление возникает в результате столкновений между носителями заряда и атомами провода, то, вероятно, столкновений будет больше в более длинном проводе. Больше столкновений означает большее сопротивление.

Во-вторых, на величину сопротивления влияет площадь поперечного сечения проводов. Более широкие провода имеют большую площадь поперечного сечения. Вода будет течь по более широкой трубе с большей скоростью, чем по узкой. Это можно объяснить меньшим сопротивлением, которое присутствует в более широкой трубе.Таким же образом, чем шире провод, тем меньше будет сопротивление прохождению электрического заряда. Когда все другие переменные одинаковы, заряд будет течь с большей скоростью через более широкие провода с большей площадью поперечного сечения, чем через более тонкие провода.

Третья переменная, которая, как известно, влияет на сопротивление потоку заряда, — это материал, из которого сделан провод. Не все материалы одинаковы с точки зрения их проводящей способности. Некоторые материалы являются лучшими проводниками, чем другие, и обладают меньшим сопротивлением потоку заряда.Серебро — один из лучших проводников, но никогда не используется в проводах бытовых цепей из-за своей стоимости. Медь и алюминий являются одними из наименее дорогих материалов с подходящей проводящей способностью, позволяющей использовать их в проводах бытовых цепей. На проводящую способность материала часто указывает его удельное сопротивление . Удельное сопротивление материала зависит от электронной структуры материала и его температуры. Для большинства (но не для всех) материалов удельное сопротивление увеличивается с повышением температуры.В таблице ниже приведены значения удельного сопротивления для различных материалов при температуре 20 градусов Цельсия.

Материал

Удельное сопротивление

(Ом • метр)

Серебро

1,59 х 10 -8

Медь

1.7 х 10 -8

Золото

2,2 х 10 -8

Алюминий

2,8 х 10 -8

Вольфрам

5,6 х 10 -8

Утюг

10 х 10 -8

Платина

11 х 10 -8

Свинец

22 х 10 -8

Нихром

150 х 10 -8

Углерод

3. 5 х 10 -5

Полистирол

10 7 — 10 11

Полиэтилен

10 8 — 10 9

Стекло

10 10 — 10 14

Твердая резина

10 13

Как видно из таблицы, существует широкий диапазон значений удельного сопротивления для различных материалов.Материалы с более низким удельным сопротивлением обладают меньшим сопротивлением потоку заряда; они лучшие дирижеры. Материалы, показанные в последних четырех строках вышеприведенной таблицы, обладают таким высоким удельным сопротивлением, что их даже нельзя рассматривать как проводники.

Посмотрите!

Используйте виджет Resistivity of a Material , чтобы найти удельное сопротивление данного материала. Введите название материала и нажмите кнопку Submit , чтобы узнать его удельное сопротивление.

Математическая природа сопротивления

Сопротивление — это числовая величина, которую можно измерить и выразить математически. Стандартной метрической единицей измерения сопротивления является ом, представленный греческой буквой омега -. Электрическое устройство с сопротивлением 5 Ом будет представлено как R = 5 . Уравнение, представляющее зависимость сопротивления ( R ) проводника цилиндрической формы (например,, провод) от влияющих на него переменных равно

, где L представляет длину провода (в метрах), A представляет площадь поперечного сечения провода (в метрах 2 ) и представляет удельное сопротивление материала (в Ом • метр). В соответствии с вышеизложенным, это уравнение показывает, что сопротивление провода прямо пропорционально длине провода и обратно пропорционально площади поперечного сечения провода. Как показано в уравнении, знание длины, площади поперечного сечения и материала, из которого изготовлен провод (и, следовательно, его удельного сопротивления), позволяет определить сопротивление провода.

Расследовать!

Резисторы — один из наиболее распространенных компонентов в электрических цепях. На большинстве резисторов нанесены цветные полосы или полосы. Цвета отображают информацию о значении сопротивления.Возможно, вы работаете в лаборатории и вам нужно знать сопротивление резистора, используемого в лаборатории. Используйте виджет ниже, чтобы определить значение сопротивления по цветным полосам.

Проверьте свое понимание

1. В бытовых цепях часто используются провода двух разной ширины: 12-го и 14-го калибра. Проволока 12-го калибра имеет диаметр 1/12 дюйма, а проволока 14-го калибра — 1/14 дюйма.Таким образом, провод 12-го калибра имеет более широкое сечение, чем провод 14-го калибра. Схема на 20 ампер, используемая для настенных розеток, должна быть подключена с использованием провода 12-го калибра, а цепь на 15 ампер, используемая для цепей освещения и вентиляторов, должна быть подключена с помощью провода 14-го калибра. Объясните физику, лежащую в основе такого электрического кода.

2. Основываясь на информации, изложенной в вопросе выше, объясните риск, связанный с использованием провода 14-го калибра в цепи, которая будет использоваться для питания 16-амперной пилы.

3. Определите сопротивление медного провода 12 калибра длиной 1 милю. Дано: 1 миля = 1609 метров и диаметр = 0,2117 см.

4. Два провода — A и B — круглого сечения, имеют одинаковую длину и изготовлены из одного материала. Тем не менее, сопротивление провода A в четыре раза больше, чем у провода B.Во сколько раз диаметр проволоки B больше диаметра проволоки A?

Сопротивление проводника — Energy Education

Рис. 1. Нить накаливания загорается из-за сопротивления проводящего провода. [1]

Сопротивление проводника — это свойство проводника при определенной температуре, и оно определяется как величина сопротивления протеканию электрического тока через проводящую среду. [2] Сопротивление проводника зависит от площади поперечного сечения проводника, длины проводника и его удельного сопротивления. Важно отметить, что электрическая проводимость и удельное сопротивление обратно пропорциональны, а это означает, что чем больше проводимость, тем меньше сопротивление.

Сопротивление проводника можно рассчитать при температуре 20 ° C с помощью: [3]

[математика] \ R = \ frac {\ rho L} {A} [/ математика]

где:

  • [math] R [/ math] — сопротивление в омах (Ом)
  • [math] \ rho [/ math] — удельное сопротивление материала в омметрах (Ом · м)
  • [math] L [/ math] — длина проводника в метрах (м)
  • [math] A [/ math] — площадь поперечного сечения проводника в метрах в квадрате (м 2 )

Эта формула говорит нам, что сопротивление проводника прямо пропорционально [математике] \ rho [ / math] и [math] L [/ math], и обратно пропорционально [math] A [/ math]. Поскольку сопротивление некоторого проводника, например отрезка провода, зависит от столкновений внутри самого провода, сопротивление зависит от температуры. С повышением температуры сопротивление провода увеличивается, так как столкновения внутри провода увеличиваются и «замедляют» протекание тока. Величина изменения определяется температурным коэффициентом. [4] Положительный температурный коэффициент приводит к увеличению сопротивления с повышением температуры, тогда как отрицательный температурный коэффициент приводит к уменьшению сопротивления с повышением температуры.Поскольку проводники обычно демонстрируют повышенное удельное сопротивление с повышением температуры, они имеют положительный температурный коэффициент. Наиболее распространенные типы резисторов — это переменные резисторы и постоянные резисторы.

Используя сопротивление проводника, можно создать свет в лампе накаливания. В лампочке накаливания есть проволочная нить определенной длины и ширины, обеспечивающая определенное сопротивление. Если это сопротивление правильное, ток, протекающий через провод, замедляется ровно настолько, без остановки из-за слишком большого сопротивления, что нить накала нагревается до точки, в которой она начинает светиться. [5]

Подробнее о сопротивлении проводника см. HyperPhysics.

PhET: Сопротивление в проводе

Университет Колорадо любезно разрешил нам использовать следующую симуляцию Фета. Изучите моделирование, чтобы увидеть, как изменяется сопротивление проводника в зависимости от геометрии и удельного сопротивления:

Для дальнейшего чтения

Для получения дополнительной информации см. Соответствующие страницы ниже:

Список литературы

Сопротивление в проводниках

  • Изучив этот раздел, вы сможете:
  • • Рассчитайте размеры проводника.
  • • Опишите влияние длины и площади поперечного сечения на сопротивление проводника.

Как размеры проводника влияют на его сопротивление

Проводник — это любой материал, который пропускает через него электрический ток. Способность любого проводника в электрической цепи пропускать ток оценивается по его электрическому СОПРОТИВЛЕНИЮ. Сопротивление — это способность противодействовать прохождению электрического тока.Напряжение — это электрическая сила, которая заставляет ток течь через проводник, но чем больше значение сопротивления любого проводника, тем меньше тока будет течь при любом конкретном значении приложенного напряжения. Сопротивление проводника в основном зависит от трех факторов:

Рис. 1.3.1 Расчет размеров проводника

1. ДЛИНА проводника.

2. ПЛОЩАДЬ ПОПЕРЕЧНЯ кондуктора.

3. МАТЕРИАЛ, из которого изготовлен проводник.

Поскольку сопротивление в более длинных проводниках больше, чем в более коротких, то:

СОПРОТИВЛЕНИЕ (R) ПРОПОРЦИОНАЛЬНО ДЛЯ ДЛИНЫ (L)

и записывается как R ∝ L (∝ означает пропорционально . ..)

Следовательно, чем длиннее проводник, тем больше сопротивление и, следовательно, меньше тока.

Также, поскольку сопротивление меньше в проводниках с большой площадью поперечного сечения:

СОПРОТИВЛЕНИЕ (R) ОБРАТНО ПРОПОРЦИОНАЛЬНО ПОПЕРЕЧНОЙ ПЛОЩАДИ (A)

, который записывается как R ∝ 1 / A (или R ∝ A -1 ).

Чем больше площадь поперечного сечения, тем больше тока может протекать по проводнику, поэтому тем ниже значение сопротивления проводника.

Круглые проводники

Если проводник имеет круглое поперечное сечение, площадь круга можно определить по формуле:

π r 2 Где π = 3,142 и r — радиус окружности.

Если поперечное сечение проводника квадратное или прямоугольное, площадь поперечного сечения проводника все же можно определить, просто умножив ширину на высоту.Большинство проводников, используемых в кабелях и т. Д., Конечно, имеют круглое поперечное сечение.

Материал, из которого изготовлен проводник, также влияет на его сопротивление, величина которого зависит от СОПРОТИВЛЕНИЯ материала, описанного в Модуле 1.4 резисторов и схем.

Сопротивление

| электроника | Британника

Узнайте, как сопротивление влияет на поток электронов в электрической цепи.

В каждой электрической цепи есть некоторое сопротивление потоку электрического тока, даже в материалах, которые являются хорошими проводниками.

Encyclopædia Britannica, Inc. Посмотреть все видео по этой статье

сопротивление , в электричестве, свойство электрической цепи или части цепи, которая преобразует электрическую энергию в тепловую энергию в противодействии электрическому току. Сопротивление включает столкновения заряженных частиц с током с неподвижными частицами, составляющими структуру проводников. Сопротивление часто считается локализованным в таких устройствах, как лампы, нагреватели и резисторы, в которых оно преобладает, хотя оно характерно для каждой части цепи, включая соединительные провода и линии электропередачи.

Рассеяние электрической энергии в виде тепла, даже если оно небольшое, влияет на величину электродвижущей силы или управляющего напряжения, необходимого для создания заданного тока в цепи. Фактически, электродвижущая сила В (измеренная в вольтах) в цепи, деленная на ток I (ампер), протекающий через эту цепь, количественно определяет величину электрического сопротивления R. Точнее, R = V / I. Таким образом, если 12-вольтовая батарея постоянно пропускает двухамперный ток по длине провода, этот провод имеет сопротивление шесть вольт на ампер или шесть Ом.Ом — это общепринятая единица электрического сопротивления, эквивалентная одному вольту на ампер и обозначаемая заглавной греческой буквой омега (Ом). Сопротивление провода прямо пропорционально его длине и обратно пропорционально его площади поперечного сечения. Сопротивление также зависит от материала проводника. См. Удельное сопротивление .

Сопротивление проводника или элемента схемы обычно увеличивается с повышением температуры. При охлаждении до крайне низких температур некоторые проводники имеют нулевое сопротивление.В этих веществах, называемых сверхпроводниками, продолжают течь токи после снятия приложенной электродвижущей силы.

Величина, обратная сопротивлению, 1/ R, называется проводимостью и выражается в единицах обратного сопротивления, называемых mho.

Последний раз редактировал и обновлял эту статью Роберт Керли.

Удельное сопротивление | Физика проводников и изоляторов

Расчет сопротивления проводов

Номинальная допустимая нагрузка проводника — это грубая оценка сопротивления, основанная на потенциальной опасности возникновения пожара по току.Однако мы можем столкнуться с ситуациями, когда падение напряжения, вызванное сопротивлением проводов в цепи, вызывает другие проблемы, кроме предотвращения возгорания. Например, мы можем проектировать схему, в которой напряжение на компоненте является критическим и не должно опускаться ниже определенного предела. В этом случае падение напряжения из-за сопротивления провода может вызвать техническую проблему, хотя оно находится в безопасных (пожарных) пределах допустимой нагрузки:

Если нагрузка в указанной выше цепи не выдерживает напряжения ниже 220 В при напряжении источника 230 В, то лучше убедиться, что проводка не упадет более чем на 10 вольт по пути.Если считать как питающие, так и обратные проводники этой цепи, остается максимально допустимое падение в 5 вольт по длине каждого провода. Используя закон Ома (R = E / I), мы можем определить максимально допустимое сопротивление для каждого отрезка провода:

Мы знаем, что длина каждого куска провода составляет 2300 футов, но как определить величину сопротивления для определенного размера и длины провода? Для этого нам понадобится другая формула:

Эта формула связывает сопротивление проводника с его удельным сопротивлением (греческая буква «ро» (ρ), которая похожа на строчную букву «p»), его длиной («l») и поперечным сечением. площадь сечения («А»).Обратите внимание, что с переменной длины в верхней части дроби значение сопротивления увеличивается по мере увеличения длины (аналогия: труднее протолкнуть жидкость по длинной трубе, чем по короткой), и уменьшается по мере увеличения площади поперечного сечения ( аналогия: жидкость легче течет по толстой трубе, чем по тонкой). Удельное сопротивление является константой для типа рассчитываемого материала проводника.

Удельное сопротивление нескольких проводящих материалов можно найти в следующей таблице.Внизу таблицы мы находим медь, уступающую только серебру по низкому удельному сопротивлению (хорошей проводимости):

Удельное сопротивление при 20 градусах Цельсия
Материал Элемент / Сплав (Ом-смил / фут) (мкОм-см)
нихром Сплав 675 112,2
Нихром В Сплав 650 108,1
Манганин Сплав 290 48. 21
Константан Сплав 272,97 45,38
Сталь * Сплав 100 16,62
Платина Элемент 63,16 10,5
Утюг Элемент 57,81 9,61
Никель Элемент 41,69 6,93
цинк Элемент 35.49 5,90
молибден Элемент 32,12 5,34
Вольфрам Элемент 31,76 5,28
Алюминий Элемент 15,94 2,650
Золото Элемент 13,32 2,214
Медь Элемент 10,09 1.678
Серебро Элемент 9,546 1,587

* = Стальной сплав с содержанием железа 99,5%, углерода 0,5%

Обратите внимание, что значения удельного сопротивления в приведенной выше таблице даны в очень странной единице «Ом-см-мил / фут» (Ом-см-мил / фут). Эта единица измерения указывает, какие единицы мы ожидаем использовать в формуле сопротивления ( R = ρl / A). В этом случае эти значения удельного сопротивления предназначены для использования, когда длина измеряется в футах, а площадь поперечного сечения измеряется в круглых милах.

Метрической единицей измерения удельного сопротивления является ом-метр (Ом-м) или ом-сантиметр (Ом-см), с 1,66243 x 10 -9 Ом-метра на Ом-см-мил / фут (1,66243 x 10 -7 Ом-см на Ом-см-дюйм). В столбце таблицы Ом-см цифры фактически масштабированы как мкОм-см из-за их очень малых величин. Например, железо указано как 9,61 мкОм-см, что может быть представлено как 9,61 x 10 -6 Ом-см.

При использовании единицы измерения удельного сопротивления Ом-метр в формуле R = ρl / A длина должна быть в метрах, а площадь — в квадратных метрах.При использовании единицы Ом-сантиметр (Ом-см) в той же формуле длина должна быть в сантиметрах, а площадь — в квадратных сантиметрах.

Все эти единицы измерения удельного сопротивления действительны для любого материала (Ом-см / фут, Ом-м или Ом-см). Тем не менее, можно предпочесть использовать Ω-cmil / ft при работе с круглым проводом, площадь поперечного сечения которого уже известна в круглых милах. И наоборот, при работе с шиной нестандартной формы или изготовленной по индивидуальному заказу шиной, вырезанной из металлического материала, когда известны только линейные размеры длины, ширины и высоты, более подходящими могут быть единицы измерения удельного сопротивления Ом-метр или Ом-см.

Решение

Возвращаясь к нашей примерной схеме, мы искали провод с сопротивлением 0,2 Ом или меньше на длине 2300 футов. Предполагая, что мы собираемся использовать медный провод (самый распространенный тип производимого электрического провода), мы можем настроить нашу формулу следующим образом:

Алгебраически решая относительно A, мы получаем значение 116 035 круговых милов. Ссылаясь на нашу таблицу размеров сплошных проводов, мы обнаруживаем, что проволока «двойной длины» (2/0) с длиной 133 100 см является достаточной, тогда как следующий меньший размер, «одинарная проводка» (1/0) с длиной 105 500 см слишком мала. .Имейте в виду, что ток в нашей цепи составляет скромные 25 ампер. Согласно нашей таблице допустимой токовой нагрузки для медного провода на открытом воздухе, достаточно было бы провода калибра 14 (если речь идет о , а не о при возгорании). Однако с точки зрения падения напряжения провод 14 калибра был бы совершенно неприемлемым.

Ради интереса, давайте посмотрим, как провод 14 калибра повлияет на характеристики нашей силовой цепи. Глядя на нашу таблицу размеров проводов, мы обнаруживаем, что проволока калибра 14 имеет площадь поперечного сечения 4 107 круглых милов.Если мы по-прежнему используем медь в качестве материала для проволоки (хороший выбор, если только мы не на действительно богаты и не можем позволить себе 4600 футов серебряной проволоки 14-го калибра!), То наше удельное сопротивление все равно будет 10,09 Ом-см · дюйм / фут. :

Помните, что это 5,651 Ом на 2300 футов медного провода калибра 14, и что у нас есть два участка по 2300 футов во всей цепи, поэтому каждый отрезок провода в цепи имеет сопротивление 5,651 Ом:

Полное сопротивление проводов нашей схемы равно 2 умноженным на 5. 651 или 11,301 Ом. К сожалению, это — слишком большое сопротивление для , чтобы обеспечить ток в 25 ампер при напряжении источника 230 вольт. Даже если бы сопротивление нагрузки было 0 Ом, сопротивление нашей проводки 11,301 Ом ограничило бы ток цепи до 20,352 ампер! Как видите, «небольшое» сопротивление провода может иметь большое значение в характеристиках схемы, особенно в силовых цепях, где токи намного выше, чем обычно встречаются в электронных схемах.

Давайте рассмотрим пример проблемы сопротивления для отрезка сборной шины, изготовленной по индивидуальному заказу.Предположим, у нас есть кусок цельного алюминиевого стержня шириной 4 см, высотой 3 см и длиной 125 см, и мы хотим рассчитать сквозное сопротивление по длине (125 см). Во-первых, нам нужно определить площадь поперечного сечения стержня:

Нам также необходимо знать удельное сопротивление алюминия в единицах измерения, соответствующих данному применению (Ом-см). Из нашей таблицы удельных сопротивлений мы видим, что это 2,65 x 10 -6 Ом-см. Установив нашу формулу R = ρl / A, мы имеем:

Как видите, из-за большой толщины шины имеет очень низкое сопротивление по сравнению со стандартными размерами проводов, даже при использовании материала с большим удельным сопротивлением.

Процедура определения сопротивления шины принципиально не отличается от процедуры определения сопротивления круглого провода. Нам просто нужно убедиться, что площадь поперечного сечения рассчитана правильно и что все единицы соответствуют друг другу, как должны.

ОБЗОР:

  • Сопротивление проводника увеличивается с увеличением длины и уменьшается с увеличением площади поперечного сечения, при прочих равных условиях.
  • Удельное сопротивление («ρ») — это свойство любого проводящего материала, показатель, используемый для определения сквозного сопротивления проводника данной длины и площади в этой формуле: R = ρl / A
  • Удельное сопротивление материалов дается в единицах Ом-смил / фут или Ом-метр (метрическая система). Коэффициент преобразования между этими двумя единицами составляет 1,66243 x 10 -9 Ом-метров на Ом-см-дюйм / фут или 1,66243 x 10 -7 Ом-см на Ом-см-дюйм / фут.
  • Если падение напряжения в цепи критично, перед выбором сечения проводов необходимо произвести точный расчет сопротивления проводов.

СВЯЗАННЫЕ РАБОЧИЕ ЛИСТЫ:

10.4: Сопротивление и сопротивление — Physics LibreTexts

Что движет током? Мы можем думать о различных устройствах, таких как батареи, генераторы, розетки и т. Д., Которые необходимы для поддержания тока.Все такие устройства создают разность потенциалов и называются источниками напряжения. Когда источник напряжения подключен к проводнику, он прикладывает разность потенциалов В , которая создает электрическое поле. Электрическое поле, в свою очередь, воздействует на свободные заряды, вызывая ток. Величина тока зависит не только от величины напряжения, но и от характеристик материала, через который протекает ток. Материал может сопротивляться потоку зарядов, и мера того, насколько материал сопротивляется потоку зарядов, известна как удельное сопротивление .Это удельное сопротивление грубо аналогично трению между двумя материалами, которые сопротивляются движению.

Удельное сопротивление

Когда к проводнику прикладывается напряжение, создается электрическое поле \ (\ vec {E} \), и заряды в проводнике ощущают силу, создаваемую электрическим полем. Плотность тока \ (\ vec {J} \), которая получается, зависит от электрического поля и свойств материала. Эта зависимость может быть очень сложной. В некоторых материалах, включая металлы при данной температуре, плотность тока приблизительно пропорциональна электрическому полю.В этих случаях плотность тока можно смоделировать как

\ [\ vec {J} = \ sigma \ vec {E}, \]

, где \ (\ sigma \) — это электрическая проводимость . Электропроводность аналогична теплопроводности и является мерой способности материала проводить или передавать электричество. 2} {V / m} = \ dfrac {A} {V \ cdot m}.{-1} \).

Электропроводность — это внутреннее свойство материала. Другим неотъемлемым свойством материала является удельное сопротивление или удельное электрическое сопротивление . Удельное сопротивление материала — это мера того, насколько сильно материал противостоит прохождению электрического тока. Символ удельного сопротивления — строчная греческая буква ро, \ (\ rho \), а удельное сопротивление — величина, обратная удельной электропроводности:

.

\ [\ rho = \ dfrac {1} {\ sigma}. \]

Единицей измерения удельного сопротивления в системе СИ является ом-метр \ ((\ Omega \ cdot m \).Мы можем определить удельное сопротивление через электрическое поле и плотность тока.

\ [\ rho = \ dfrac {E} {J}. \]

Чем больше удельное сопротивление, тем большее поле необходимо для создания заданной плотности тока. Чем ниже удельное сопротивление, тем больше плотность тока, создаваемого данным электрическим полем. {-1}\)ConductorsSemiconductors [1]Insulators»>

Материалы, перечисленные в таблице, разделены на категории проводников, полупроводников и изоляторов на основе широких групп удельного сопротивления. У проводников наименьшее удельное сопротивление, а у изоляторов наибольшее; полупроводники имеют промежуточное удельное сопротивление. Проводники имеют разную, но большую плотность свободных зарядов, тогда как большинство зарядов в изоляторах связаны с атомами и не могут двигаться. Полупроводники являются промежуточными, имеют гораздо меньше свободных зарядов, чем проводники, но обладают свойствами, из-за которых количество свободных зарядов сильно зависит от типа и количества примесей в полупроводнике.Эти уникальные свойства полупроводников находят применение в современной электронике, о чем мы поговорим в следующих главах.

Пример \ (\ PageIndex {1} \): плотность тока, сопротивление и электрическое поле для токоведущего провода

Рассчитайте плотность тока, сопротивление и электрическое поле 5-метрового медного провода диаметром 2,053 мм (калибр 12), по которому проходит ток \ (I — 10 \, мА \).

Стратегия

Мы можем рассчитать плотность тока, сначала найдя площадь поперечного сечения провода, которая равна \ (A = 3.{-5} \ dfrac {V} {m}. \ End {align *} \]

Значение

Исходя из этих результатов, неудивительно, что медь используется для проводов, проводящих ток, потому что сопротивление довольно мало. Обратите внимание, что плотность тока и электрическое поле не зависят от длины провода, но напряжение зависит от длины.

Упражнение \ (\ PageIndex {1} \)

Медные провода обычно используются для удлинителей и домашней электропроводки по нескольким причинам.2} \). Третья важная характеристика — пластичность. Пластичность — это мера способности материала вытягиваться в проволоку и мера гибкости материала, а медь обладает высокой пластичностью. Подводя итог, можно сказать, что проводник является подходящим кандидатом для изготовления проволоки, по крайней мере, с тремя важными характеристиками: низким удельным сопротивлением, высокой прочностью на разрыв и высокой пластичностью. Какие еще материалы используются для электромонтажа и в чем преимущества и недостатки?

Ответ

Серебро, золото и алюминий используются для изготовления проволоки.Все четыре материала обладают высокой проводимостью, серебро — самой высокой. Все четыре элемента легко сворачиваются в проволоку и обладают высоким пределом прочности на разрыв, хотя и не таким высоким, как медь. Очевидным недостатком золота и серебра является их стоимость, но серебряные и золотые провода используются для специальных применений, таких как провода для динамиков. Золото не окисляется, улучшая связи между компонентами. У алюминиевых проводов есть свои недостатки. Алюминий имеет более высокое удельное сопротивление, чем медь, поэтому требуется больший диаметр, чтобы соответствовать сопротивлению на длину медных проводов, но алюминий дешевле, чем медь, поэтому это не является серьезным недостатком.Алюминиевая проволока не обладает такой высокой пластичностью и прочностью на разрыв, как медная, но пластичность и предел прочности находятся в допустимых пределах. Есть несколько проблем, которые необходимо решить при использовании алюминия, и следует соблюдать осторожность при выполнении соединений. Алюминий имеет более высокий коэффициент теплового расширения, чем медь, что может привести к ослаблению соединений и возможной опасности возгорания. Окисление алюминия не проводит и может вызвать проблемы. При использовании алюминиевых проводов необходимо использовать специальные методы, а компоненты, такие как электрические розетки, должны быть рассчитаны на прием алюминиевых проводов.

ФЭТ

Просмотрите это интерактивное моделирование, чтобы увидеть, как площадь поперечного сечения, длина и удельное сопротивление провода влияют на сопротивление проводника. Отрегулируйте переменные с помощью ползунков и посмотрите, станет ли сопротивление меньше или больше.

Температурная зависимость удельного сопротивления

Вернувшись к таблице \ (\ PageIndex {1} \), вы увидите столбец с надписью «Температурный коэффициент». Удельное сопротивление некоторых материалов сильно зависит от температуры.У некоторых материалов, таких как медь, удельное сопротивление увеличивается с повышением температуры. Фактически, в большинстве проводящих металлов удельное сопротивление увеличивается с повышением температуры. Повышение температуры вызывает повышенные колебания атомов в структуре решетки металлов, которые препятствуют движению электронов. В других материалах, таких как углерод, удельное сопротивление уменьшается с повышением температуры. Во многих материалах зависимость является приблизительно линейной и может быть смоделирована с помощью линейного уравнения:

\ [\ rho \ приблизительно \ rho_0 [1 + \ alpha (T — T_0)], \]

, где \ (\ rho \) — удельное сопротивление материала при температуре Тл , \ (\ alpha \) — температурный коэффициент материала, а \ (\ rho_0 \) — удельное сопротивление при \ (T_0 \) , обычно принимается как \ (T_0 = 20.oC \).

Обратите внимание, что температурный коэффициент \ (\ alpha \) отрицателен для полупроводников, перечисленных в таблице \ (\ PageIndex {1} \), что означает, что их удельное сопротивление уменьшается с увеличением температуры. Они становятся лучшими проводниками при более высоких температурах, потому что повышенное тепловое перемешивание увеличивает количество свободных зарядов, доступных для переноса тока. Это свойство уменьшения \ (\ rho \) с температурой также связано с типом и количеством примесей, присутствующих в полупроводниках.

Сопротивление

Теперь рассмотрим сопротивление провода или компонента. Сопротивление — это мера того, насколько сложно пропустить ток через провод или компонент. Сопротивление зависит от удельного сопротивления. Удельное сопротивление является характеристикой материала, используемого для изготовления провода или другого электрического компонента, тогда как сопротивление является характеристикой провода или компонента.

Для расчета сопротивления рассмотрим участок проводящего провода с площадью поперечного сечения A , длиной L и удельным сопротивлением \ (\ rho \).Батарея подключается к проводнику, обеспечивая разность потенциалов \ (\ Delta V \) на нем (рисунок \ (\ PageIndex {1} \)). Разность потенциалов создает электрическое поле, которое пропорционально плотности тока, согласно \ (\ vec {E} = \ rho \ vec {J} \).

Рисунок \ (\ PageIndex {1} \): потенциал, обеспечиваемый батареей, прикладывается к сегменту проводника с площадью поперечного сечения \ (A \) и длиной \ (L \).

Величина электрического поля на отрезке проводника равна напряжению, деленному на длину, \ (E = V / L \), а величина плотности тока равна току, деленному на поперечную. площадь сечения \ (J = I / A \).Используя эту информацию и вспоминая, что электрическое поле пропорционально удельному сопротивлению и плотности тока, мы можем видеть, что напряжение пропорционально току:

\ [\ begin {align *} E & = \ rho J \\ [4pt] \ dfrac {V} {L} & = \ rho \ dfrac {I} {A} \\ [4pt] V & = \ left (\ rho \ dfrac {L} {A} \ right) I. \ end {align *} \]

Определение: Сопротивление

Отношение напряжения к току определяется как сопротивление \ (R \):

\ [R \ Equiv \ dfrac {V} {I}. \]

Сопротивление цилиндрического сегмента проводника равно удельному сопротивлению материала, умноженному на длину, разделенную на площадь:

\ [R \ Equiv \ dfrac {V} {I} = \ rho \ dfrac {L} {A}. \]

Единица измерения сопротивления — Ом \ (\ Омега \). Для заданного напряжения чем выше сопротивление, тем ниже ток.

Резисторы

Обычным компонентом электронных схем является резистор. Резистор можно использовать для уменьшения протекания тока или обеспечения падения напряжения.На рисунке \ (\ PageIndex {2} \) показаны символы, используемые для резистора в принципиальных схемах цепи. Два широко используемых стандарта для принципиальных схем предоставлены Американским национальным институтом стандартов (ANSI, произносится как «AN-см.») И Международной электротехнической комиссией (IEC). Обе системы обычно используются. Мы используем стандарт ANSI в этом тексте для его визуального распознавания, но отметим, что для более крупных и сложных схем стандарт IEC может иметь более четкое представление, что упрощает чтение.

Рисунок \ (\ PageIndex {2} \): символы резистора, используемые в принципиальных схемах. (а) символ ANSI; (b) символ IEC.

Зависимость сопротивления материала и формы от формы

Резистор можно смоделировать как цилиндр с площадью поперечного сечения A и длиной L , сделанный из материала с удельным сопротивлением \ (\ rho \) (рисунок \ (\ PageIndex {3} \)) . Сопротивление резистора \ (R = \ rho \ dfrac {L} {A} \)

Рисунок \ (\ PageIndex {3} \): Модель резистора в виде однородного цилиндра длиной L и площадью поперечного сечения A .Его сопротивление потоку тока аналогично сопротивлению трубы потоку жидкости. Чем длиннее цилиндр, тем больше его сопротивление. Чем больше его площадь поперечного сечения A , тем меньше его сопротивление.

Наиболее распространенным материалом для изготовления резистора является углерод. Углеродная дорожка намотана на керамический сердечник, к нему прикреплены два медных вывода. Второй тип резистора — это металлопленочный резистор, который также имеет керамический сердечник. Дорожка сделана из материала оксида металла, который имеет полупроводниковые свойства, аналогичные углеродным.Опять же, в концы резистора вставляются медные провода. Затем резистор окрашивается и маркируется для идентификации. Резистор имеет четыре цветные полосы, как показано на рисунке \ (\ PageIndex {4} \).

Рисунок \ (\ PageIndex {4} \): Многие резисторы напоминают рисунок, показанный выше. Четыре полосы используются для идентификации резистора. Первые две цветные полосы представляют собой первые две цифры сопротивления резистора. Третий цвет — множитель. Четвертый цвет обозначает допуск резистора.{-5} \, \ Omega \), а сверхпроводники вообще не имеют сопротивления при низких температурах. Как мы видели, сопротивление зависит от формы объекта и материала, из которого он состоит.

Сопротивление объекта также зависит от температуры, поскольку \ (R_0 \) прямо пропорционально \ (\ rho \). Для цилиндра мы знаем \ (R = \ rho \ dfrac {L} {A} \), поэтому, если L и A не сильно изменяются с температурой, R имеет такую ​​же температурную зависимость, как \ ( \ rho \).(Исследование коэффициентов линейного расширения показывает, что они примерно на два порядка меньше типичных температурных коэффициентов удельного сопротивления, поэтому влияние температуры на L и A примерно на два порядка меньше, чем на \ (\ rho \).) Таким образом,

\ [R = R_0 (1 + \ alpha \ Delta T) \ label {Tdep} \]

— это температурная зависимость сопротивления объекта, где \ (R_0 \) — исходное сопротивление (обычно принимаемое равным \ (T = 20,00 ° C \), а R — сопротивление после изменения температуры \ (\ Дельта Т \).oC \).

Многие термометры основаны на влиянии температуры на сопротивление (Рисунок \ (\ PageIndex {5} \)). Один из наиболее распространенных термометров основан на термисторе, полупроводниковом кристалле с сильной температурной зависимостью, сопротивление которого измеряется для определения его температуры. Устройство небольшое, поэтому быстро приходит в тепловое равновесие с той частью человека, к которой прикасается.

Рисунок \ (\ PageIndex {5} \): Эти знакомые термометры основаны на автоматическом измерении сопротивления термистора в зависимости от температуры.oC) \ right) \\ [5pt] & = 4.8 \, \ Omega \ end {align *} \]

Значение

Обратите внимание, что сопротивление изменяется более чем в 10 раз, когда нить накала нагревается до высокой температуры, а ток через нить накала зависит от сопротивления нити и приложенного напряжения. Если нить накаливания используется в лампе накаливания, начальный ток через нить накала при первом включении лампы будет выше, чем ток после того, как нить накала достигнет рабочей температуры.

Упражнение \ (\ PageIndex {2} \)

Тензодатчик — это электрическое устройство для измерения деформации, как показано ниже. Он состоит из гибкой изолирующей основы, поддерживающей рисунок из проводящей фольги. Сопротивление фольги изменяется по мере растяжения основы. Как меняется сопротивление тензодатчика? Влияет ли тензодатчик на изменение температуры?

Ответ

Рисунок фольги растягивается по мере растяжения основы, а дорожки фольги становятся длиннее и тоньше.Поскольку сопротивление рассчитывается как \ (R = \ rho \ dfrac {L} {A} \), сопротивление увеличивается по мере того, как дорожки из фольги растягиваются. При изменении температуры меняется и удельное сопротивление дорожек фольги, изменяя сопротивление. Один из способов борьбы с этим — использовать два тензодатчика, один используется в качестве эталона, а другой — для измерения деформации. Два тензодатчика поддерживают при постоянной температуре

Сопротивление коаксиального кабеля

Длинные кабели иногда могут действовать как антенны, улавливая электронные шумы, которые являются сигналами от другого оборудования и приборов.Коаксиальные кабели используются во многих случаях, когда требуется устранение этого шума. Например, их можно найти дома через кабельное телевидение или другие аудиовизуальные соединения. Коаксиальные кабели состоят из внутреннего проводника с радиусом \ (r_i \), окруженного вторым внешним концентрическим проводником с радиусом \ (r_0 \) (рисунок \ (\ PageIndex {6} \)). Пространство между ними обычно заполнено изолятором, например полиэтиленовым пластиком. Между двумя проводниками возникает небольшой ток радиальной утечки.Определите сопротивление коаксиального кабеля длиной L .

Рисунок \ (\ PageIndex {6} \): Коаксиальные кабели состоят из двух концентрических проводников, разделенных изоляцией. Они часто используются в кабельном телевидении или других аудиовизуальных средствах связи.

Стратегия

Мы не можем использовать уравнение \ (R = \ rho \ dfrac {L} {A} \) напрямую. Вместо этого мы смотрим на концентрические цилиндрические оболочки толщиной dr и интегрируем.

Решение

Сначала мы находим выражение для \ (dR \), а затем интегрируем от \ (r_i \) до \ (r_0 \),

\ [\ begin {align *} dR & = \ dfrac {\ rho} {A} dr \\ [5pt] & = \ dfrac {\ rho} {2 \ pi r L} dr, \ end {align *} \]

Объединение обеих сторон

\ [\ begin {align *} R & = \ int_ {r_i} ^ {r_0} dR \\ [5pt] & = \ int_ {r_i} ^ {r_0} \ dfrac {\ rho} {2 \ pi r L } dr \\ [5pt] & = \ dfrac {\ rho} {2 \ pi L} \ int_ {r_i} ^ {r_0} \ dfrac {1} {r} dr \\ [5pt] & = \ dfrac {\ rho} {2 \ pi L} \ ln \ dfrac {r_0} {r_i}. \ end {align *} \]

Значение

Сопротивление коаксиального кабеля зависит от его длины, внутреннего и внешнего радиусов, а также удельного сопротивления материала, разделяющего два проводника. Поскольку это сопротивление не бесконечно, между двумя проводниками возникает небольшой ток утечки. Этот ток утечки приводит к ослаблению (или ослаблению) сигнала, передаваемого по кабелю.

Упражнение \ (\ PageIndex {3} \)

Сопротивление между двумя проводниками коаксиального кабеля зависит от удельного сопротивления материала, разделяющего два проводника, длины кабеля и внутреннего и внешнего радиуса двух проводников.Если вы разрабатываете коаксиальный кабель, как сопротивление между двумя проводниками зависит от этих переменных?

Ответ

Чем больше длина, тем меньше сопротивление. Чем больше удельное сопротивление, тем выше сопротивление. Чем больше разница между внешним радиусом и внутренним радиусом, то есть чем больше соотношение между ними, тем больше сопротивление. Если вы пытаетесь максимизировать сопротивление, выбор значений для этих переменных будет зависеть от приложения.Например, если кабель должен быть гибким, выбор материалов может быть ограничен.

Phet: Цепь батарейного резистора

Просмотрите это моделирование, чтобы увидеть, как приложенное напряжение и сопротивление материала, через который протекает ток, влияют на ток через материал. Вы можете визуализировать столкновения электронов и атомов материала, влияющие на температуру материала.

Авторы и авторство

  • Сэмюэл Дж.Линг (Государственный университет Трумэна), Джефф Санни (Университет Лойола Мэримаунт) и Билл Мобс со многими авторами. Эта работа лицензирована OpenStax University Physics в соответствии с лицензией Creative Commons Attribution License (4.0).

Сопротивление проводника

Хотя можно использовать провод любого размера или значения сопротивления, слово «проводник» обычно относится к материалам, которые обладают низким сопротивлением току, а слово «изолятор» описывает материалы, которые обладают высоким сопротивлением току. .Между проводниками и изоляторами нет четкой разделительной линии; при определенных условиях все типы материалов проводят ток. Материалы, обеспечивающие сопротивление току на полпути между лучшими проводниками и самыми плохими проводниками (изоляторами), иногда называют «полупроводниками» и находят наибольшее применение в области транзисторов.

Лучшие проводники — это материалы, в основном металлы, которые обладают большим количеством свободных электронов; И наоборот, изоляторы — это материалы, в которых мало свободных электронов.Лучшие проводники — серебро, медь, золото и алюминий; но некоторые неметаллы, такие как углерод и вода, могут использоваться в качестве проводников. Такие материалы, как резина, стекло, керамика и пластмассы, являются настолько плохими проводниками, что их обычно используют в качестве изоляторов. Ток в некоторых из этих материалов настолько мал, что обычно считается нулевым. Единица измерения сопротивления называется ом. Символ ома — греческая буква омега (Ω). В математических формулах заглавная буква «R» обозначает сопротивление.Сопротивление проводника и приложенное к нему напряжение определяют количество ампер тока, протекающего по проводнику. Таким образом, сопротивление 1 Ом ограничивает ток до 1 ампера в проводнике, к которому приложено напряжение 1 вольт.

Факторы, влияющие на сопротивление

  1. Сопротивление металлического проводника зависит от типа материала проводника. Было указано, что некоторые металлы обычно используются в качестве проводников из-за большого количества свободных электронов на их внешних орбитах.Медь обычно считается лучшим доступным материалом для проводников, поскольку медная проволока определенного диаметра обеспечивает меньшее сопротивление току, чем алюминиевая проволока того же диаметра. Однако алюминий намного легче меди, и по этой причине, а также по соображениям стоимости, алюминий часто используется, когда важен весовой коэффициент.
  2. Сопротивление металлического проводника прямо пропорционально его длине. Чем больше длина провода данного размера, тем больше сопротивление.На рисунке 12-41 показаны два проводника разной длины. Если электрическое давление 1 вольт приложено к двум концам проводника длиной 1 фут, а сопротивление движению свободных электронов предполагается равным 1 Ом, ток ограничивается 1 ампер. Если провод того же размера удвоить в длину, те же электроны, приведенные в движение под действием приложенного 1 вольта, теперь обнаруживают удвоенное сопротивление; следовательно, ток уменьшается вдвое. Рисунок 12-41. Сопротивление зависит от длины проводника.
  3. Сопротивление металлического проводника обратно пропорционально площади поперечного сечения. Эта область может быть треугольной или даже квадратной, но обычно круглой. Если площадь поперечного сечения проводника увеличивается вдвое, сопротивление току уменьшается вдвое. Это верно из-за увеличенной площади, в которой электрон может перемещаться без столкновения или захвата атомом. Таким образом, сопротивление изменяется обратно пропорционально площади поперечного сечения проводника.
  4. Четвертым основным фактором, влияющим на сопротивление проводника, является температура.Хотя некоторые вещества, такие как углерод, демонстрируют снижение сопротивления при повышении температуры окружающей среды, большинство материалов, используемых в качестве проводников, увеличивают сопротивление при повышении температуры. Сопротивление некоторых сплавов, таких как константан и манганин ™, очень мало изменяется при изменении температуры. Величина увеличения сопротивления 1-омного образца проводника на один градус повышения температуры выше 0 ° по Цельсию (C), принятого стандарта, называется температурным коэффициентом сопротивления.Для каждого металла это разные значения. Например, для меди это значение составляет примерно 0,00427 Ом. Таким образом, медный провод, имеющий сопротивление 50 Ом при температуре 0 ° C, имеет увеличение сопротивления на 50 × 0,00427 или 0,214 Ом на каждый градус повышения температуры выше 0 ° C. Температурный коэффициент сопротивления необходимо учитывать там, где наблюдается заметное изменение температуры проводника во время работы. Доступны графики с указанием температурных коэффициентов сопротивления для различных материалов.На Рис. 12-42 показана таблица «удельного сопротивления» некоторых распространенных электрических проводников.

Рисунок 12-42. Таблица удельного сопротивления.

Сопротивление материала определяется четырьмя свойствами: материалом, длиной, площадью и температурой. Первые три свойства связаны следующим уравнением при T = 20 ° C (комнатная температура):

Сопротивление и связь с размером провода

Круглые проводники (провода / кабели)

Поскольку известно, что Сопротивление проводника прямо пропорционально его длине, и если нам дано сопротивление единичной длины провода, мы можем легко вычислить сопротивление любой длины провода из того же материала, имеющего тот же диаметр.Кроме того, поскольку известно, что сопротивление проводника обратно пропорционально его площади поперечного сечения, и если нам дано сопротивление отрезка провода с единичной площадью поперечного сечения, мы можем вычислить сопротивление такой же длины. из проволоки из того же материала любой площади сечения. Следовательно, если мы знаем сопротивление данного проводника, мы можем рассчитать сопротивление для любого проводника из того же материала при той же температуре. Из отношения:

Также можно записать:

Если у нас есть проводник длиной 1 метр (м) с площадью поперечного сечения 1 (миллиметр) мм 2 и сопротивлением 0 .017 Ом, каково сопротивление 50 м провода из того же материала, но с площадью поперечного сечения 0,25 мм 2 ?

В то время как единицы СИ обычно используются при анализе электрических цепей, электрические проводники в Северной Америке все еще производятся с использованием стопы в качестве единицы длины и мил (одна тысячная дюйма) в качестве единицы диаметра. Прежде чем использовать уравнение R = (ρ × l) ⁄A для расчета сопротивления проводника данного американского калибра проводов (AWG), площадь поперечного сечения в квадратных метрах должна быть определена с использованием коэффициента преобразования 1 mil = 0. .0254 мм. Самая удобная единица длины проволоки — стопа. В соответствии с этими стандартами единицей измерения является мил-фут. Таким образом, проволока имеет единичный размер, если она имеет диаметр 1 мил и длину 1 фут.

В случае использования медных проводов мы избавляемся от утомительных вычислений с помощью таблицы, показанной на Рисунке 12-43. Обратите внимание, что размеры поперечного сечения, указанные в таблице, таковы, что каждое уменьшение на один номер датчика равняется 25-процентному увеличению площади поперечного сечения.Из-за этого уменьшение трех калибровочных номеров означает увеличение площади поперечного сечения примерно на 2: 1. Аналогичным образом, изменение десяти калибровочных номеров проводов представляет собой изменение площади поперечного сечения 10: 1 — кроме того, при удвоении площади поперечного сечения проводника сопротивление уменьшается вдвое. Уменьшение сечения проводов на три сечения снижает сопротивление проводника заданной длины вдвое.

Рисунок 12-43.

Добавить комментарий

Ваш адрес email не будет опубликован. Обязательные поля помечены *